Sunteți pe pagina 1din 93

Training & Placement

Pondicherry Engineering College


Puducherry

Preparatory Material 1. Numerical Reasoning 2. Analytical Reasoning 3. Logical Reasoning 4. Verbal Reasoning

Training & Placement

1. Numerical Reasoning 1.1 Problems on Numbers


1.1.1. Concept: Numbers are broadly classified into Real and Imaginary Numbers. The Real Numbers consist of Integers, Whole Numbers and Natural Numbers. Integers are the set of all NonFractional Numbers lying between and i.e., ... 4, 3, 2, 1, 0, 1, 2, . Natural Numbers are the set of all NonFractional Numbers from 1 to infinity i.e. 1, 2, 3, . Whole Numbers are the set of all Natural Numbers and Zero 0, 1, 2, . The Imaginary Numbers are generally written in the form of a + bi, where a and b are Real Numbers and i is the Imaginary Number and its value is 1. Operation on Complex Numbers is similar to that on Real Numbers. Let a + bi and c + di be two Complex Numbers. Then the various arithmetic operations are: Addition : (a + bi) + (c + di) = [(a + c) + (b + d) i] Example : (3 + 4i) + (4 3i) = [(4 + 3) + (4 3) i] = 7 + i Subtraction : (a + bi) (c + di) = [(a c) + (b d) i] Example : (3 + 4i) (4 3i) = [(3 4) + (4 3) i] = 1 + 7i Multiplication : (a + bi) * (c + di) = ac + adi + cbi+ bdi2 = (ac bd) + (ad + bc) i Example : (3 + 4i) (4 3i) = (12 (12)) + (9+16) i = 24 + 7i The Numbers that have two factors Only and Exactly the number itself and 1 are all Prime Numbers. Natural Numbers greater than 1, which are not Prime, are known as Composite Numbers. A Common Factor of two or more numbers is a number which divides each of them exactly. For example, 4 is a Common Factor of 8 and 12. H.C.F. (Highest Common Factor) is the greatest number that divides each one of them exactly. For example, 6 is the Highest Common Factor of 12, 18 and 24. It is also called Greatest Common Divisor (G.C.D.) The Least Common Multiple (L.C.M.) of two or more given numbers is the least or lowest number which is exactly divisible by each of them. For example, consider the two numbers 12 and 18. Multiples of 12 are 12, 24, 36, 48, 72 and Multiples of 18 are 18, 36, 54, 72, . Common Multiples are 36 and 72. Therefore, Least Common Multiple of 12 and 18 is 36. Arithmetic Progression (A.P.) is a sequence of numbers in which the difference between any two successive numbers is always constant. If the Ratio of any two successive terms is equal throughout the sequence, then the sequence is said to be in a Geometric Progression (G.P.)

Pondicherry Engineering College

Training & Placement Tips: 1. To square any number ending with 5, let the number be A5. (A5)2 = A (A + 1)25. Ex: 252 = (2 *(2 + 1)) 25 = 625. (A = 2, join 25 instead of multiplying). 2. A number is divisible by 3, if the Sum of the digits in the number is divisible by3. Ex: 3792 is divisible by 3 since 3+7+9+2 = 21, which is divisible by 3. 3. A number is divisible by 4, if the number formed by the last two digits is divisible by 4 or both the last two digits are zero. Ex: The number 2616 is divisible by 4 since 16, the number formed by the last two digits are divisible by 4. 4. A number is divisible by 6, if the number is even and Sum of its digits is divisible by 3. Ex: The number 4518 is divisible by 6 since it is even and Sum of its digits 4+5+1+8 = 18, is divisible by 3. 5. A number is divisible by 8, if the number formed by the last 3 digits is divisible by 8. Ex: The number 41784 is divisible by 8 as the number formed by the last 3 digits is Divisible by 8. 6. A number is divisible by 9 if the Sum of its digits is divisible by 9. Ex: The number 819 is divisible by 9 as the sum of the digits, 8+1+9 = 18, is divisible by 9 7. A number is divisible by 12 if it is divisible by both 3 and 4. Ex: The number 24 is divisible by 3 and 4 is also divisible by 12 8. In Comparison of Fractions, if the Denominators of the Fractions are same, the largest fraction is the one whose Numerator is the largest. Ex: 7/8 > 5/8 > 3/8. 9. If the Numerators of the Fractions are same, the largest fraction is the one whose Denominator is the smallest. Ex: 5/2 > 5/7 > 5/9. 10. If a Composite number C, which can be expressed as C = am * bn * cp.where a, b, care all Prime Factors and m, n, p are Positive Integers, the number of Factors is equal to (m + 1) (n + 1) (p + 1) Pondicherry Engineering College 2

Training & Placement Ex: The total numbers of Factors of 576 can be found by writing 576 = (24 * 24) = (23 * 3) * (23 * 3) = (26 * 32) Therefore, the number of Factors = (6 + 1)* (2 + 1) = 21. 11. To find the L.C.M. and H.C.F. of Fractions: L.C.M. of Fractions = (L.C.M. of Numerators) / (H.C.F. of Denominators) H.C.F. of Fractions = (H.C.F. of Numerators) / (L.C.M. of Denominators) 1.1.2. Formulae: 1. Dividend = (Quotient * Divisor) + Remainder 2. In A.P, to find the nth term, Tn = a + (n 1) d, where a is the first term, and d is the difference. 3. To find the Sum of n numbers, Sn = n / 2 [2a + (n 1) d] or Sn = n* (a + l)/2 4. In G.P, to find the nth term Tn = a rn1. a(rn 1) if r >1 r1 6. L.C.M * H.C.F. = Product of the numbers L.C.M. = Product of the numbers H.C.F. Problem 1: 5. Sum Sn = How many three digit numbers are divisible by 6 in all? Solution: Formula: Tn = a + (n 1) d Three digit number divisible by 6 range from 102, 108, 114 996 Tn = 996, a = 102, d = 6 996 = 102 + (n 1) 6 Problem 2: A number when divided by 342 gives a Remainder 47. When the same number is divided by 18 what would be the Remainder? Solution: Dividend = (Divisor * Quotient) + Remainder = 342Q + 47 = 19 * 18Q + 18 * 2 + 11 = 18(19Q + 2) + 11 Remainder is 11. Pondicherry Engineering College 3 Hence n = 150

Training & Placement Problem 3: Find the Greatest 5-digit number which is exactly divisible by 463. Solution: The Greatest 5-digit number is 99999. Dividing this number by 463, the Remainder is 454. So, the required number = 99999 454 = 99545. 1.1.3. Problems to Solve: 1. A certain number when divided by 899 gives a Remainder 63. Remainder when the same number is divided by 29? 2. How many numbers between 11 and 90 are divisible by 7? 3. The Sum of the Squares of two numbers is 3341 and the difference of their Squares is 891. Find the number. 4. In a two-digit number, the Sum of the digits is 15. If 9 is added to the number, the digits are reversed. Find the numbers. 5. What is the Maximum number of Slices you can obtain by cutting a cake with only 4 cuts? 6. Two numbers are such that the Ratio between them is 4:7. If each is increased by 4, the Ratio becomes 3:5. Find the Largest number. 7. The Denominator of a Fraction is 3 more than the Numerator. If 4 is added to both the Numerator and the Denominator, the Fraction becomes 4/5. Find the Fraction. 8. The Sum of three numbers is 136. If the Ratio between first and second is 2:3 and that between second and third is 5:3, find the second number. 9. The Product of two numbers is 120 and the Sum of their Squares is 289. Find the Sum of the numbers. 10. The Sum of the Squares of three Consecutive Odd numbers is 251. Find the numbers. What is the

1.2. Ratio and Proportion


1.2.1. Concept: Ratio is a comparison of two quantities. It is a relationship between two quantities with respect to magnitude. If a and b are two numbers, the Ratio of a to b is a/b and is denoted by a : b. The two quantities that are being compared are called terms. The first term is called Antecedent and the second term is called Consequent. 1.2.2. Types of Ratios: Duplicate Ratio: The Ratio of the Squares of two numbers is called the Duplicate Ratio of those two numbers. Ex: 32 / 42. Pondicherry Engineering College 4

Training & Placement Triplicate Ratio: The Ratio of the Cubes of two numbers is called the Triplicate Ratio of that numbers. Ex: 33 / 43 SubDuplicate Ratio: The Ratio of the Square roots of two numbers is called the Sub Duplicate Ratio of two numbers. Ex: 3/4 is the SubDuplicate Ratio of 9 / 16. Inverse Ratio: If the antecedent and consequent of a Ratio interchange their places, the new Ratio is called the Inverse Ratio of the first. Ex: 3 / 5 is the Inverse Ratio of 5 / 3. Compound Ratio: The Ratio of the Product of the antecedents to that of the consequents of two or more given Ratios is called the Compound Ratio. Ex: If 3 / 5, 4 / 5 and 5 / 7 are the given Ratios, then (3*4*5) / (5*5*7) i.e. 12 / 35 is the compound Ratio. The Equality of two Ratios is called Proportion. Ex: 3 / 5 = 4 / 10, we write 3 : 5 : : 4 : 10. If a / b = c / d, then each term of the Ratio a / b and c / d is called a Proportional. Here, a, d are known as Extremes and b, c are known as Means. If four quantities are in Proportion, then Product of Means = Product of Extremes If a : x :: x : b, x is called the Mean or Second Proportional of a, b. If a : b :: b : x, x is called the Third Proportional of a, b. If a : b :: c : x, x is called the Fourth Proportional of a, b, c.

Tips:
1. If two numbers are in the Ratio of a : b and the Sum of these numbers is x, then these numbers will be ax / (a + b) and bx / (a + b) respectively. 2. If in a Mixture of x litres, two liquids A and B are in the Ratio of a : b, then the Quantities of Liquids A and B in the Mixture will be ax / (a + b) litres and bx / (a + b) respectively. 3. If the Ratio of two numbers is a : b, then the numbers that should be added to each of the numbers in order to make this Ratio c : d is given by (ad bc) / (c d). 4. If the Ratio of two numbers is a : b, then the number that should be subtracted from each of the numbers in order to make this Ratio c : d is given by (bc ad) / (c d). 5. If the Incomes of two Persons are in the Ratio of a : b and their Expenditures are in the Ratio of c : d. If the Saving of each Person be Rs. S, then their Incomes are given by, Rs. (aS (d c)) / (ad bc) and Rs. (bS (d c)) / (ad bc) 6. If a : b = n1 : d1 and b : c = n2 : d2, then a : b : c = (n1*n2) : (d1*n2) : (d1 : d2)

Pondicherry Engineering College

Training & Placement Problem 1: Two numbers are in the Ratio of 4 : 5 and the Sum of these numbers is 27.Find the two numbers. Solution: Here, a = 4, b = 5, and x = 27. The first number = (a * x) / (a + b) = (4 * 27) / (4+ 5) = 12 The second number will be = 27 12 = 15. Problem 2: Two numbers are in the Ratio 9 : 7.If 14 is subtracted from each, the Ratio becomes 7 : 5. Find the numbers. Solution: Let the numbers be 9x and 7x. 9x 14 = 7 7x 14 5 5(9x 14) = 7(7x 14) => 45x 70 => 98 70 => x Problem 3: = = = 49x 98 49x 45x 7 Hence the numbers are 9x = 63 and 7x = 49.

Find the number that must be subtracted from the Ratio of 5 : 6 to make it equal to 2 : 3. Solution: We have a : b = 5 : 6 and c : d = 2 : 3. The required number = (bc ad) / (c d) = [(6*2) (5*3)] / (2 3) = 3. 1.2.3. Problems to Solve: 1. Rs. 49 was divided among 150 children. Each Girl got 50 paise and each Boy 25 paise. How many Boys were there? 2. Five Bananas and Four Apples cost as much as Three Bananas and Seven Apples. Find the Ratio of the cost of One Banana to that of One Apple. 3. Annual Income of A and B is in the Ratio of 5 : 4 and their Annual Expenses bear a Ratio of 4 : 3. If each of them saves Rs. 500 at the end of the Year, then find their Annual Income. 4. If 1 / 3A = 1 / 4B = 1 / 5C, then A: B: C =? 5. A, B, C and D share a Property worth Rs. 77, 500. If A : B = 3 : 2, B: C = 5 : 4 and C : D = 3 : 7, Find Share of B.

Pondicherry Engineering College

Training & Placement 6. A Man spends Rs 500 in buying 12 items of Tables and Chairs. The cost of one table is Rs. 50 and that of one chair is Rs. 40. What is the Ratio of the numbers of Chairs and Tables purchased? 7. An Alloy contains Copper and Zinc in the Ratio 7 : 3. If the alloy contains 10.5 kg Zinc, find the Quantity of Copper in the Alloy. 8. If Income of A, B and C are in the Ratio of 9 : 3 : 7 and Income of C exceeds the Income of B by Rs. 1200 then find the Income of A. 9. Find the Age of Amit and Puneet which are in the Ratio 2 : 3. After 12 years, their Ages will be in the Ratio 11 : 15. What is the age of Puneet? 10. Two numbers A and B are such that the Sum of 5% of A and 4% of B is 2/3 rd of the Sum of 6% of A and 8% of B. Find the Ratio of A : B.

1.3. Alligation or Mixture


1. 3.1. Concept: Alligation means Linking. It is a Rule to find the Ratio in which two or more Ingredients at their Respective Prices should be mixed to give a Mixture at a given Price. By using this Rule, we can also find the Mean or Average Price of a Mixture when the Prices of two or more Ingredients which may be mixed together and the Proportion in which they are mixed are given. Alligation Rule: Suppose, Rs. D per Unit be the Price of first ingredient (superior quality) mixed with another ingredient (cheaper quality) of Price Rs. C per unit to form a Mixture whose Mean Price is Rs. M per unit, then the two ingredients must be mixed in the Ratio: Quantity of Cheaper = C.P. (Cost Price) of Costlier Mean Quantity of Costlier Mean C.P. of Cheaper The two Ingredients are to be mixed in the Inverse Ratio of the Differences of their Prices and the Mean Price. The above Rule may be represented schematically as Cost of Cheaper Cost of Costlier C D

(DM) Ratio = (D M): (M C)

(MC)

Pondicherry Engineering College

Training & Placement Problem 1: Sugar at Rs. 15 per kg is mixed with Sugar at Rs. 20 per kg in the Ratio 2 : 3. Find the Price per kg of the Mixture. Solution: c (Rs.15) d (Rs.20)

m (Rs.x)

d m = 20 x From the given data,

m c = x 15

20 x = 2 . x 15 3 x = 18 Price of the Mixture = Rs. 18 per kg.

Problem 2: A merchant has 100 kg of salt, part of which he sells at 7% Profit and the rest at 17% Profit. He Gains 10% on the whole. Find the Ratio. Solution: 7 17

10 (17 10) Ratio = (17 10): (10 7) Therefore, the Ratio is 7: 3. 1.3.2. Problems to Solve: 1. In what ratio is ghee worth of Rs. 30 per Kg and ghee worth of Rs. 45 per Kg be mixed so as to get the mixture worth of Rs. 40 per Kg? 2. How many kilograms of sugar costing Rs. 9 per kg must be mixed with 27 kg of sugar costing Rs. 7 per kg so that there may be a Gain of 10% by selling the Mixture at Rs. 9.24 per kg? 3. In what Ratio two varieties of Tea, one costing Rs. 27 per Kg and the other costing Rs. 32 per Kg should be blended to produce a blended variety of Tea worth Rs. 30 per Kg? How much should be the quantity of Second Variety of Tea, if the First Variety is 60 Kg? Pondicherry Engineering College 8 (10 7)

Training & Placement 4. How much Water is to be added to 14 litres of Milk worth Rs. 5.40 a litre so that the value of the Mixture may be Rs. 4.20 a litre? 5. In what Ratio must Water be added to Spirit to Gain 10% by selling it at the cost Price? 6. Two vessels A and B contain Milk and Water mixed in the Ratio 4 : 3 and 2 : 3. In what Ratio must these Mixtures be mixed to form a new Mixture containing half Milk and half Water? 7. In an Examination out of 480 students 85% of the Girls and 70% of the Boys passed. How many Boys appeared in the Examination if the total Pass Percentage was 75%? 8. Some amount out of Rs. 7000 was lent at 6% p.a. and the remaining at 4% p.a. If the Total Simple Interest from both the Fractions in 5 Years was Rs. 1600, find the Sum lent at 6% p.a. 9. Miras Expenditure and Savings are in the Ratio 3 : 2. Her Income increases by 10%. Her Expenditure also increases by 12%. By what % does her saving increase? 10. The Average Weekly Salary of all Employees (Supervisors and Laborers) is Rs. 100. The Average Weekly Salary of all the supervisors is Rs.600 while the Average weekly Salary of all the laborers is Rs. 75. Find the number of supervisors in the factory if there are 840 laborers in it

1.4. Chain Rule


1.4.1. Concept: There are two Types of Variations 1) Direct Variation 2) Indirect Variation Direct Variation: Apples Cost (Rs.) 20 140 12 x If number of Apples is less, cost is also less. If the Variation of one thing, while increasing or decreasing, affects the other thing in the same way, then it is Direct Variation. 20 / 12 = 140 / x x = 140*12 20 = 84 Indirect Variation: Men 14 24 Days 8 x Pondicherry Engineering College

Training & Placement If number of Men increase, the Days required for the Completion of the work will be less. When one value increases, then the other value decreases. It is called Indirect Variation. 24 14 = 8 x

24* x = 8* 14 x = 8 * 14 24 x = 14 3 Problem 1: If 11.25 m of a Uniform Iron Rod weighs 42.75 kg, what will be the weight of 6 m of the same rod? Solution: Length (m) weight (kg) 11.25 42.75 6 x Since it is a Direct Proportion, x _ = 6 42.75 11.25 x = 6 * 42. 75 11.25 22.8 kg

x Problem 2:

39 Persons can repair a road in 12 Days, working 5 Hours a day. In how many Days will 30 Persons working 6 Hours a day, complete the same work? Solution: Men Days Hours 39 30 12 x 5 6

More Hours less Days (Inverse Proportion) Less Men more Days (Inverse Proportion) x 12 x x = 5 * 39 6 30 (5 *39 * 12) / (6 * 30) 13 Days.

= =

Pondicherry Engineering College

10

Training & Placement 1.4.2. Problems to Solve: 1. If 5 toys cost Rs. 234, how much will 55 Toys cost? 2. If 45 Men can do a Piece of Work in 20 hrs, in how many Hours can 15 Men do it? 3. If the Wages of 9 Men for 14 Days is Rs. 2100, then find the wages of 18 Men for 7 Days. 4. If 20 Men can build a wall of 56 meters long in 6 Days, what length of a similar wall can be built by 35 Men in 3 Days? 5. If 12 Men working 9 Hours a day can reap a field in 16 Days, in how many Days will 18 Men reap the field working 8 Hours a day? 6. 20 Men complete onethird of work in 20 Days. How many more Men should be employed to finish the rest of the work in 25 more Days? 7. In a dairy farm, 40 cows eat 40 bags of husk in 40 Days. In how many Days will one cow eat one bag of husk? 8. A contract is to be completed in 46 Days and 117 Men were asked to work 8 Hours a day. After 33 Days, 4/7 of the work is completed. How many additional Men should be employed so that the work could be completed in time, each man now working 9 Hours a day? 9. If 6 Men working 8 Hours a day earn Rs.840 per week, then how much will 9 Men working 6 Hours a day earn per week? 10. If 3 Men or 6 Boys can do a Piece of Work in 10 Days working 7 Hours a day, how many Days will it take to complete a Piece of Work twice as large with 6 Men and 2 Boys working together for 8 Hours a day?

1.5. Partnership
1.5.1. Concept: In Partnership two or more Persons carry on a Business and Share the Profits of the Business at an agreed Proportion. Persons who have entered into Partnership with one another are individually called Partners and collectively called a Firm and the name under which their Business is carried on is called the Firm Name. The Partnership may be Simple or Compound. Simple Partnership is one in which the capital of each Partner is in the Business for same time. Compound Partnership is one in which the capitals of Partners are invested for different periods. In this, a Partner can be a working Partner or Sleeping Partner. Sleeping Partner is one who invests the capital in the Business but does not actively participate in the conduction of Business. Working Partner besides investing capital takes part in running the Business. For his work, he is either paid some Salary or given a certain percent of Profit in addition. Pondicherry Engineering College 11

Training & Placement 1.5.2. Formulae: 1. If A invested Rs. x and B invested Rs. y then Ratio A : B = X : Y 2. If A invested Rs. x and after 3 months B invested Rs. y then the Share is in the Ratio A : B = (x * 12) : (y * 9). 2. If Capitals of two Partners are Rs. C1 and Rs. C2, and the total Profit is Rs. P, then Shares of the Partners in the Profits are Rs. ((C1 * P) / (C1 + C2)) and Rs. ((C2 * P) / (C1 + C2)). Tips: 1. If the Capitals of two Partners is Rs.C1 and Rs.C2 for the periods t1 and t2 respectively, then Profit of A / Profit of B = (C1*t1) / (C2*t2) 2. If three Partners invested their capitals in a Business. If the timing of their investment is in the Ratio t1: t2: t3 and their Profits are in the Ratio P1 : P2 : P3, then the Ratio of their capitals invested is (P1 / t1) : (P2 / t2) : (P3 / t3). Problem 1: Sona, Sweetha, Sumathy entered into a Partnership investing Rs.35000, Rs. 45000 and Rs. 55000 respectively. Find the respective Shares of Sona, Sweetha Sumathy if the annual Profit is Rs. 40500. Solution: Ratio Sonas Share Sweethas Share Sumathys Share Problem 2: Ramani started a Business investing Rs. 9000. After Five months, Raj joined with a capital of Rs. 8000. If at the end of the Year, they earn a Profit of Rs. 6970, then what will be the Share of Raj in the Profit? Solution: Ratio of their Share Rajs Share = = = = 9000*12: 8000*7 27 : 14 Rs. (6970*14 / 41) Rs.2380. = = = = 35000: 45000: 55000 40500 * 7 / 27 40500 * 9 / 27 40500*11 / 27 = = = = 7: 9: 11 Rs. 10500. Rs. 13500. Rs. 16500.

Pondicherry Engineering College

12

Training & Placement Problem 3: A and B start a Business with the amount of Rs.10000 and Rs.15000 respectively. After 3 months, A withdraws Rs. 2000 and after 6 months again he withdraws Rs. 4000. At the end of the Year they get the Profit of Rs. 7000. What will be their Share according to the Investment? Solution: As investment = = = Bs investment = = A: B = = As Share Bs Share = = (10000 * 3) + (8000 * 6) + (4000 * 3) 30000 + 48000 + 12000 Rs. 90000 15000 *12 Rs. 1, 80,000 90000 : 180000 1:2 7000 * 1 / 3 7000 * 2 / 3 = = Rs. 2333 Rs. 4667

1.5.3. Problems to Solve: 1. Find the amount of Profit each Partner A, B and C will get if they invest Rs. 20000, Rs. 50000 and Rs. 40000 respectively in a Business. The net Profit for the Year was Rs. 12100 which was divided in Proportion to investments made. 2. A and B enter into a Partnership with Rs. 50000 and Rs. 60000 respectively. C joins them after 6 months contributing Rs. 70000 and B leaves x months before the end of the Year. If they Share the Profit in the Ratio of 20 : 18 : 21, find the value of x. 3. A and B started a Business investing Rs. 9000 and Rs. 12000. After 6 months, B withdrew half of his investment. After a Year, if the total Profit is Rs. 4600, what would be Bs Share in it? 4. Rs. 120 is divided among X, Y and Z so that Xs Share is Rs. 20 more than Ys Share and Rs. 20 less than Zs Share. What is Ys Share? 5. Rahul started a Business with a capital of Rs. 8000. After Six months, Sanjay joined him with Investment of some Capital. If at the end of the Year each of them gets Equal Amount as Profit, how much did Sanjay invest in the Business?

6. Two Partners invest Rs. 12500 and Rs. 8500 respectively in a Business. If one Partner gets Rs.300 more than the other in the Profit, what is the Total Profit? 7. Gupta and Bansal enter into a Partnership with their capitals in the Ratio 5 : 6. At the end of 8 months, Gupta withdraws his capital. If they receive the Profits in the Ratio of 5: 9, find how long Bansals Capital was used.

Pondicherry Engineering College

13

Training & Placement 8. A and B jointly Invest Rs. 2100 and Rs. 3100 respectively in a Firm. A is an Active Partner and hence he gets 25% of Profit separately. If their Business yields them total of Rs. 1040 as Profit, what will be the Gain of each of them? 9. A began Business with Rs. 6000 and was joined afterwards by B with Rs. 8000. At the end of the Year B got onethird of the total Profit. When did B join? 10. X and Y started a Business. X invested Rs. 3000 for 4 months and Y invested Rs. 2000 for 6 months. How much should X be paid out of a total Profit of Rs. 500?

1.6. Area and Volume


1.6.1. Concept: The Area of any figure is the amount of surface enclosed within its bounding lines. Area is always expressed in square units. The Perimeter of a Geometrical Figure is the Total Length of the sides enclosing the Figure. A Closed Figure bounded by three sides is called a Triangle. A Triangle having one of its angles equal to 90 degrees is called a RightAngled Triangle. A Triangle in which all sides are equal is called an Equilateral Triangle. A Triangle whose two sides are equal is an Isosceles Triangle. A Closed Figure bounded by four sides is called a Quadrilateral. A Quadrilateral in which opposite sides are equal and parallel is called a Parallelogram. Square is a Quadrilateral with all sides equal and all the Four Angles equal to 90o. A Rectangle is a Quadrilateral with opposite sides equal and all the four angles equal to 90 o. A Rhombus is a Quadrilateral in which all sides are equal. A Trapezium is a Quadrilateral in which any two opposite sides are parallel. Distance between parallel sides is called its Height. Circle is the path traveled by a point which moves in such a way that its distance from a fixed point remains constant. The fixed point is known as the Centre and the fixed distance is called the Radius. A Solid is a figure bounded by one or more surfaces. It has Three Dimensions namely Length, Breadth or Width and Height. The Volume of any solid figure is the amount of space enclosed within its bounding spaces. It is measured in cubic units. Cuboid is a Solid Figure which has Six Rectangular Faces. It is also called as Rectangular Parallelopiped. A Right Circular Cylinder is a solid with circular ends of equal radius and the line joining their centers perpendicular to them. This is called the Axis of the Cylinder. The Length of the Axis is called the Height of the Cylinder. This can be best illustrated by rolling a rectangular paper either lengthwise or breadth-wise in a round way, i.e. a cylinder is generated by rotating a rectangle by fixing one of its sides. A Right Circular Cone is a solid obtained by rotating a RightAngled Triangle around its height.

Pondicherry Engineering College

14

Training & Placement Pythagoras Theorem: In a Right Angled Triangle, the Square of the Hypotenuse is equal to the Sum of the Squares of the other two sides. E.g.: In a Rightangled Triangle ABC Right angled at B, AC2 = AB2 + AC2 1.6.2. Formulae: 1. Area of Square = (Side) * (Side) = (Side)2 Perimeter of Square = 4*(Side) 2. Area of Rectangle = Length * Breadth Perimeter of Rectangle = 2 * (Length + Breadth) 3. Area of Triangle = * Base * Height 4. Area of Triangle = (s*(s a)* (s b)* (s c)) where S = a + b + c , a, b and c are sides of the Triangle. 2 5. Area of equilateral Triangle = 3 a2, where a is side of the Triangle. 4 3 a, where a is side of the Triangle. 2 7. Area of a circle = r2 , where r is radius of the circle. 8. Circumference of the Circle = 2 r 9. Area of Rhombus = * d1 * d2 where d1 and d2 are diagonals. 10. Area of a Parallelogram = b*h 11. Area of a Trapezium = [ * (a + b) * h] Where a and b are parallel sides and h is the height. 12. Volume of the Cube = (edge) 3 = a3. 13. Total surface area of Cube = 6 * a2 14. Diagonal of the Cube = a* 3 15. Volume of cylinder = Area of the base * height 16. Area of the curved surface = Circumference of the base * height 17. Area of the total surface = Area of the curved surface + Area of two circular ends. 18. Volume of Cone = 1/3 * ( r2h) Where h is height and r is Radius of Base. 19. Area of the largest Circle that can be inscribed in a Square of side a is a2. 4 2 20. Area of a Square inscribed in a Circle of Radius r is 2r and the side of the Square is 2 r. 21. The Area of Largest Triangle inscribed in a Semi Circle of Radius r is r2 . 6. Altitude of Equilateral Triangle =

Pondicherry Engineering College

15

Training & Placement Tips: 1. If the Length and the Breadth of a Rectangle are increased by x% and y% respectively, then the Area of Rectangle will increase by (x + y + xy / 100) %.

2. If the Length of a rectangle is increased by x%, then its breadth will have to be decreased by (100x / (100 + x)) % in order to maintain the same area of rectangle. 3. If each of the defining dimensions or sides of any twodimensional figure is changed by x%, its area changes by x*(2 + x/100) %. 4. If all the sides of a Quadrilateral are increased (or decreased) by x%, its diagonals also increase (or decrease) by x%. 5. If the Ratio of the areas of two Squares be a : b, then the Ratio of their sides, Ratio of their Perimeters and the Ratio of their diagonals, each will be in the Ratio a : b. 6. If the Length and Breadth of a room are l and b respectively, and a Carpet of Width w is used to cover the floor, then the required Length of the Carpet is (l * b)/ w. 7. If the Length and Breadth of a room are l and b respectively, then the least number of Square tiles required to cover the floor = (l*b) / (H.C.F (l, b)). 8. A Rectangular garden l m long and b m broad is surrounded by a path w m wide. The Area of the Path is given by 2w*(l + b + 2w) sq.m. 9. If all three measuring dimensions of a sphere, cuboid, cube, cylinder or cone are increased or decreased by x%, y% and z% respectively, then the volume of the figure will increase or decrease by (x + y + z + ((xy + yz + xyz) / 100) + xyz/100)% 10. If a Sphere of Radius R is melted to form smaller Spheres each of Radius r, then the number of small Spheres = Big Spheres Volume / Small Spheres Volume (R / r)3. Problem 1: What is the Cost of planting the field in the form of the Triangle whose Base is 2.8 m and Height 3.2 m at the rate of Rs.100 / m2? Solution: Area of Triangular field Cost Problem 2: Area of a Rhombus is 850 cm2. If one of its Diagonal is 34 cm, find the Length of the other Diagonal. = = = = * 3.2 * 2.8 m2 4.48 m2 Rs.100 * 4.48 Rs. 448.

Pondicherry Engineering College

16

Training & Placement Solution: Area of a Rhombus 850 Therefore, d2 Problem 3: The Radius and Height of a Cylinder are increased by 10% and 20% respectively. Find the Percentage increase in Surface Area. Solution: Let the increase in Radius x be 10% and the increase in Height y be 20%. Percentage increase in Surface Area is = = 1.6.3. Problems to Solve: 1. If the Base of an Isosceles Triangle is 10 cm and the Length of equal sides is 13 cm, find its area. 2. How many tiles of 20 cm by 10 cm will be needed to cover the floor of a room 25 m long and 16 m wide? 3. The Length and Breadth of a Rectangle are increased by 20% and 5% respectively. Find the Percentage increase in its area. 4. Find the Area of a Trapezium having Parallel sides 65 m and 44 m and their separation being 20 m. 5. The Diameter of a Circle is 105 cm less than the Circumference. What is the Diameter of a Circle? 6. The Area of the Parallelogram is 72 sq.cm and its Height is 12 cm. Find the Base of the Parallelogram. 7. The Diameter of a Wheel is 2 cm. If it rolls forward covering 10 Revolutions, find the Distance traveled by it. 8. Find the Area of an Equilateral Triangle each of whose sides measure 6 cm. 9. A Rectangular Carpet has an Area of 120 sq.m. and a Perimeter of 46 m. Find the Length of its Diagonal. Pondicherry Engineering College 17 (x + y + (xy / 100)) % (10 + 20 + ((10*20) / 100)) % = 32 % = = 850 / 17 50 cm = = = * d1 * d2 * 34 * d2 17 * d2

Training & Placement

10.A Copper Sphere of Diameter 18 cm is drawn into a Wire of Diameter 4 mm. Find
the Length of the Wire.

1.7. Probability
1.7.1. Concept: Probability is a concept which measures Uncertainties. Probability is the chance of occurring of a certain Event. It is expressed quantitatively. Probability has its origin in the problems dealing with games of chance such as gambling, coin tossing, dice throwing and playing cards. The possible outcomes of a trial are called Events. Events are generally denoted by capital letters. Events are said to be Equally likely if there is no reason to expect anyone in preference to other. In Simple Events, we consider the Probability of happening or non happening of single events. In the case of Compound Events we consider the occurrence of two or more Events. If the occurrence of an event precludes or rules out the happening of all the other Events in the same experiment, it is called Mutually Exclusive Event. Two Events A and B are said to be Independent if the occurrence of one does not affect the Probability of the occurrence of the other. The number of combinations of n objects taken r at a time (r <= n) is denoted by C (n, r) or nCr and is denoted as nCr = n! / (r! * (n r)!) If A and B are two Events associated with sample space S, then P (A B) is the Probability that either A or B or both the Events occur. P (A B) is the Probability that A and B occur simultaneously. 1.7.2 Formulae: 1. P (E) = n(E) n(S) E Event S Sample space

2. P (A B) = P (A) + P (B) P (A B), where P (A) is the Probability of Event A and P (B) is the Probability of Event B. 3. If the Events are Mutually Exclusive, then P (A P(A B) = 0 for Mutually Exclusive Events. Tips: 1. P (E) + P (E1) = 1 2. Independent Events P (A B) = P (A) *P (B) B) = P (A) + P (B), because

3. Probability of Occurrence of an Event is always positive. Pondicherry Engineering College

18

Training & Placement 4. Probability of Occurrence of an Event is from zero to one. 5. Probability of Occurrence of an impossible Event is 0. Problem 1: If two Dice are thrown simultaneously, what is the Probability that the first Dice shows up 5 and the second Dice does not show up 5? Solution: We want only 5 on first Dice and any number other than 5 on the second Dice, the number of favorable cases are 5 viz., (5, 1) (5, 2) (5, 3) (5, 4) (5, 6) Hence the Probability = 5 / 36. Problem 2: A bag contains 2 Green, 6 Blue, and 3 Black balls. If a Ball is drawn at random, what is the Probability that it is not a Black ball? Solution: One ball can be drawn out of the 11 balls from the bag in 11C1 ways. A ball other than a black ball (2 + 6 = 8) can be drawn in 8C1 ways. Hence, the Probability that the ball drawn is not black is 8C1 / 11C1 = 8 / 11. Problem 3: From a well shuffled pack of 52 Cards, a Card is drawn at random. Find the Probability that it is either a Heart or a Queen. Solution: P(A) = Getting a Heart Card; P(B) = Getting a Queen Card. P(A) = 13 / 52; P(B) = 4 / 52; P (A B) = 1 / 52. Required Probability, P(A B) = P(A) + P(B) P (A B) = (13 / 52) + (4 / 52) 1 / 52 = 4 / 13. 1.7.3. Problems to Solve: 1. Two Dice are thrown. Find the Probability of getting an Odd number on one Dice and a multiple of three on the other

Pondicherry Engineering College

19

Training & Placement 2. A man and his wife appear for an interview for two vacancies for the same post. The Probability of husbands selection is 1/7 and the Probability of wifes selection is 1/ 5. What is the Probability that only one of them is selected? 3. A Card is drawn from pack of 100 Cards numbered 1 to 100. Find the Probability of drawing a number which is a Square. 4. Two Dice are thrown. Find the Probability that the total score is a Prime Number. 5. A Card is drawn from a pack of 52 cards. Find the Probability of getting a King of Club or a Queen of Heart. 6. A Coin is tossed Three times. Find the chance that Head and Tail show alternatively. 7. One bag contains 4 White and 2 Black balls. Another bag contains 3 White and 5 Black balls. One ball is drawn from each bag; Find the Probability that both are White. 8. Five Letters are placed at random in Five Addressed Envelopes. Find the Probability that all the Letters are not dispatched in the Right Envelopes. 9. The Probability that a man will be alive for 25 Years and hence is 0.3 and the Probability that his wife will be alive for 25 Years and hence is 0.4. Find the Probability that 25 Years hence both will be alive. 10. Find the Probability of getting a total of 7 or 11 in a simultaneous throw of two Dice.

1.8. Time and Work


1.8.1. Concept: The Time required for the Completion and the number of Men engaged for a Project is Inversely Proportional to each other, i.e. the more number of Men involved, the lesser is the time required to finish a job. If A can do a Piece of Work in n Days, then at a Uniform Rate of Working, A will finish 1 / nth work in 1 day. A, B and C can do a Piece of Work in T1, T2 and T3 Days, respectively. If they have worked for D1, D2 and D3 Days respectively, then Amount of Work done by A Amount of Work done by B Amount of Work done by C Also, Amount of Work done (A, B and C together) = = = = = D1 / T1 D2 / T2 D3 / T3 D1 / T1 + D2 / T2 + D3 / T3 1 (if the work is complete)

Pondicherry Engineering College

20

Training & Placement Tips: 1. If A can do a Piece of Work in n Days and B can do the same Piece of Work in m Days then Work will be done in [m*n / (m + n)]Days, if both Work together. 2. If A, B and C while working alone, can complete a Work in X, Y and Z Days respectively, then they will together complete the Work in X*Y*Z / (XY + YZ + ZX). 3. Two Persons A and B, working together, can complete a Piece of Work in X Days. If A, Working alone, can complete the work in Y Days, then B, working alone will complete the work in X*Y / (Y X) Days. If A works Thrice that of B then Ratio of Work done by A and B is 3 : 1. If A can complete a / b part of Work in X Days, then c / d part of the work will be done in (b*c*X) / (a*d) Days. If a Men and b Women can do a Piece of Work in n Days, then c Men and d Women can do the work in (n*a*b) / [(b*c) + (a*d)].

4. 5.

6.

Problem 1: Worker A takes 15 hrs to complete a job. Worker B takes 30 hrs to do the same job. How long should it take both A and B, working together but independently, to do the same job? Solution: = 1 / 15 = 1 / 30 = (1 / 15) + (1 / 30) = (2 + 1) / 30 = 1 / 10 Both A and B will finish the Work in 10 Days. Problem 2: A can do a Piece of Work in 6 Days, and B can do it in 12 Days. What time will they require to do it working together? Solution: Part of the work done by A in one day = 1/6 Part of the work done by B in one day = 1 / 12 Part of the work done by A and B in one day = 1 / 6 + 1 / 12 = 1/4 Time required by A and B together to finish the work = 4 Days. As 1 Hours work Bs 1 Hours work (A + B)s 1 Hours work

Pondicherry Engineering College

21

Training & Placement Problem 3: 12 Men or 15 Women can do a work in 14 Days. In how many Days, 7 Men and 5 Women would complete the work? Solution: Here, A = 12, B = 15, N = 14 C = 7 and D = 5. Required Number of Days 1.8.2. Problems to Solve: 1. A can do a Piece of Work in 30 Days while B can do it in 40 Days. If A and B Work Together, in how many Days can they complete the work? 2. 10 Men can complete a Piece of Work in 15 Days and 15 Women can complete the same work in 12 Days. If all the 10 Men and 15 Women work together, in how many Days will the work get completed? 3. A can finish a work in 18 Days and B can do the same work in 15 Days. B worked for 10 Days and left the job. In how many Days, A alone can finish the remaining work? 4. X can do a Piece of Work in 40 Days. He works at it for 8 Days and then Y finishes it in 16 Days. How long will they together take to complete the work? 5. 4 Men and 6 Women can complete a work in 8 Days, while 3 Men and 7 Women can complete it in 10 Days. In how many Days will 10 Women complete it? 6. A and B can finish a job in 12 Days while A, B and C can finish it in 8 Days. How long will it take for C alone to complete the work? 7. 5 Men can complete a work in 2 Days, 4 Women can complete the same work in 3 Days and 5 children can do it in 3 Days. If 1 man, 1 Woman and 1 child work together, in how many Days will they complete the work? 8. A can do a Piece of Work in 8 Days. A undertook to do it for Rs. 320. With the help of B he finishes the work in 6 Days. What will be the Share of B? 9. A can do a piece of work in 9 Days. If B is 50% more efficient than A, then in how many Days can B do the same work? 10. Two Men undertake to do a Piece of Work for Rs. 200. One alone can do it in 6 Days and the other in 8 Days. With the help of a Boy they finish it in 3 Days. How much is the Share of the Boy? Pondicherry Engineering College 22

= = =

n*a*b / [(b*c) + (a*d)] 14*12*15 / ((15*7) + (12*5)) 168 / 11 Days.

Training & Placement

1.9. Pipes and Cisterns


1.9.1. Concept: Pipes are connected to a Tank or Cistern and are used to fill or empty the Tank. Inlet is a Pipe connected to a Tank or Cistern for filling it. Outlet is a Pipe connected to a Tank or Cistern for emptying it. The concept is similar to that of Time and work. If an inlet can completely fill the Tank in X Hours, the part of the Tank filled in 1 Hour is 1 / X. If an outlet can empty the full Tank in Y Hours, the part of the Tank emptied in 1Hour is 1 / Y. If both inlet and outlet are open, net part of the Tank filled in 1Hour = 1 / X 1 / Y. Tips: 1. If an inlet can completely fill the empty Tank in X Hours, the part of the Tank filled in 1 Hour is 1 / X. If an outlet can empty the full Tank in Y Hours, the part of the Tank emptied in 1 Hour is 1 / Y. If inlet and outlet are open, net part of the Tank filled in 1 Hour = 1 / X 1 / Y. Two Pipes A and B can fill a Cistern in X and Y Hours respectively while working alone. If both the Pipes are opened together, then the time taken to fill the Cistern is (X*Y) / (X + Y) Hours. Two Pipes A and B can fill a Cistern in X Hours and Y Hours, respectively. There is also an outlet C. If all the three Pipes are opened together, the Tank is full in Z Hours. The time taken by C to empty the full Tank is (X*Y*Z) / (XZ + YZ XY) Hours. A Cistern has a leak which can empty it in X Hours. A Pipe which allows Y litres of Water per Hour into the Cistern is turned on and now the Cistern is emptied in Z Hours. The capacity of the Cistern can be found as (X*Y*Z) / (Z X) litres.

2.

3. 4.

5.

6.

Problem 1: A tap can fill a Tank in 6 Hours. After half the Tank is filled three more similar taps are opened. What is the total time taken to fill the Tank completely? Solution: Time taken to fill the half Tank = 3 Hours. Part filled in 1 Hour = 1/6 Part filled by the four taps in 1 Hour = (4* 1 / 6) =2 / 3 Remaining part (1 1 / 2) =1/2 2 / 3: 1 / 2 = 1: x or x = (1 / 2* 1* 3 / 2) = 3 / 4 hrs i.e. All the four taps need 45 minutes more to fill the remaining Tank. So, totally 3 Hours 45 minutes is the time taken to fill the Tank completely Pondicherry Engineering College 23

Training & Placement Problem 2: 12 buckets of Water fill a Tank when the capacity of each Tank is 13.5 litres. How many buckets will be needed to fill the same Tank, if the capacity of each bucket is 9 litres? Solution: Capacity of Tank Capacity of each bucket Number of buckets needed Problem 3: A Cistern is filled in 9 Hours and it takes 10 Hours when there is a leak in its bottom. If the Cistern is full, in what time shall the leak empty it? Solution: Work done in 1 Hour by the leak and the filling Pipe Work done by the leak in 1 Hour = = = Hence the leak can empty it in 90 Hours. 1.9.2. Problems to Solve: 1. Three Pipes A, B and C can fill a Cistern in 10, 12 and 15 Hours respectively while working alone. If all the three Pipes are opened together, find the time taken to fill the Cistern. A Tap can fill a Tank in 25 minutes and another can empty it in 50 minutes. Find whether the Tank will be filled up or emptied first and in how many minutes? A Tap can fill a bath in 20 minutes and another tap can fill it in 30 minutes. Amit opens both the taps simultaneously. When the bath should have been full, he finds that the waste Pipe was open. He then closes the waste Pipe and in another 4 minutes the bath is full. What time will the waste Pipe empty it? A Cistern can be filled by two Pipes in 20 and 30 minutes respectively. Both Pipes are opened. When must the first Pipe be turned off so that the Cistern may be filled in 10 more minutes? Three Pipes A, B and C can fill a Cistern in 6 hrs. After working together for 2 hrs, C is closed and A and B fill the Cistern in 8 hrs. Find the time in which the Cistern can be filled by Pipe C. Two Pipes A and B together can fill a Cistern in 24 min and 32 min respectively. Both Pipes being open, Find when the first Pipe must be turned off so that the Cistern may be just filled in 16 minutes. 1/9 1 / 9 1/10 1 / 90 = = = = (12 x 13.5) litres 162 litres 9 litres (162 / 9) = 18

2.

3.

4.

5.

6.

Pondicherry Engineering College

24

Training & Placement 7. A leak in the bottom of a Tank can empty the full Tank in 8 hrs. An inlet Pipe fills Water at the rate of 6 litres a minute. When the Tank is full, the inlet is opened and due to leak the Tank is empty in 12 Hours. How many litres does the Cistern hold? Two Pipes A and B separately fill a Tank in 6 hrs and 8 hrs respectively. Both the Pipes are opened together, but 1 hrs after the start the Pipe A is turned off. How much time will it take to fill the Tank? If two Pipes function simultaneously, the reservoir will be filled in 12 Hours. One Pipe fills the reservoir 10 Hours faster than the other. How many Hours does the faster Pipe take to fill the reservoir?

8.

9.

10. A reservoir is provided by two Pipes A and B. A can fill the reservoir 5 Hours faster than B. If both together fill the reservoir in 6 Hours, in how many Hours the reservoir will be filled by A alone?

1.10. Interest
1.10.1. Concept: When a Person A borrows some money from another Person B, then A has to pay certain amount to B for the use of this money. This amount paid by A is called Interest. The total amount of money borrowed by A from B is called the Principal. The money paid back to B, which comprises of the Principal and the Interest is called the Amount. The Interest is usually charged according to a specified term, which is expressed as some percent of the principal and is called the Rate of interest for fixed period of time. Interest can be of two types: 1. Simple Interest (S.I) 2. Compound Interest (C.I)

When the interest is payable on the principal only, it is called Simple Interest. For example, Simple Interest on Rs. 100 at 5% per annum will be Rs.5 each Year, that is, at the end of one Year total amount will be Rs. 105. At the end of second Year it will be Rs. 110 and so on. Compound Interest is the method, where the interest for each period is added to the Principal before interest is calculated for the next period. With this method, the Principal grows as the interest is added to it. 1.10.2. Formulae:

Simple Interest:
Amount = P+I I = (P*N*R) / 100 Where P Principal, N Period Rate of interest, I interest It can be written as R = (I x 100) / (P*N)

Pondicherry Engineering College

25

Training & Placement

Compound Interest:
Amount A = P* (1 + R / 100) N CI = [Amount Principal]. Example: Difference between C.I and S.I for 2 Years C.I S.I = P*[R / 100] 2 Example: Difference between C.I and S.I for 3 Years C.I S.I = P*[(R / 100)3 + 3 (R / 100)2]

Tips (Simple Interest):


1. If a certain Sum in T Years at R% per annum amounts to Rs. A, the Sum will be P = ((100*A) / (100 + (R*T))) 2. The annual payment that will discharge a debt of Rs. A due in T Years at R% per annum is Annual payment = Rs.(100*A) / ((100*T) + (R*T*(T 1)) / 2) 3. If a certain Sum of money becomes n times itself in T Years at Simple Interest, then the Rate of Interest per annum is R = 100*(n 1) / T %. 4. If a certain Sum of money becomes n times itself in T Years at a Simple Interest, then the time T1 in which it will become m times itself is given by T1 = (m 1)* T / (n 1) 5. If a debt of Rs. Z is paid in n number of installments and if the value of each installment is Rs. A, then the borrowed amount is given by, Z = (n*a) + [(R*a) / (100*b)] * [n*(n1) / 2] 6. If a certain Sum of money P lent out at SI amounts to A1 in T1 Years and A2 in T2 Years, then P = (A1T2 A2T1) / (T2 T1). 7. If an amount P1 lent at Simple Interest rate of R1% per annum and another amount P2 at Simple Interest rate of R2% per annum, Rate of Interest for the whole Sum is R = (P1R1 + P2R2) / (P1 + P2).

Tips (Compound Interest):


8. When the Rates of Interest are different for different Years, say R1, R2, R3 per cent for first, second and third Year respectively, then Amount = P*[1+ (R1/100)]* [1+ (R2/100)]* [1+ (R3/100)]

Problem 1: Find the S.I on a Principal of Rs.1000 at an Interest Rate of 4% per annum for a period of 4 Years. Pondicherry Engineering College 26

Training & Placement Solution: The formula for S.I Where P is principal R is rate of interest N is time period S.I Problem 2: = = = = = PNR / 100 Rs.1000 4% 4 Years 1000*4*4 / 100 = Rs.160

Find the Simple Interest on Rs. 5200 for 2 Years at 6% per annum. Solution: Here, P = Rs.5200, T = 2 Years and R = 6% Therefore, Simple Interest = P*R*T/100 = 5200*6*2/100 = Rs. 624. Problem 3: Mahesh borrowed Rs. 3000 from his friend Suresh at 15 per cent per annum for 3 Years. Find the Interest and Money returned by Mahesh to Suresh. Solution: Here, P = Rs.3000, R = 15% per annum, T = 3 Years Interest, I = = = = = = P*R*T / 100 3000*15*3 / 100 Rs.1350. P + I. Rs.3000 + Rs.1350 Rs.4350.

Amount, A

Problems to Solve: 2. A man borrowed Rs.8000 at the rate of 12% S.I. and lent the whole Sum to another Person at the rate of 15%. What will be the Gain after 7 Years? In what time, can we get Rs.1200 as S.I. on Rs.5000 at the rate of 8%? Find the rate percent, if the Compound Interest on Rs.15625 for 3 Years is Rs.1951? Mr. Krishna received a loan at 13% per annum S.I. After 4 Years he returned the principal and interest. If he returned Rs. 9120, what will be the Principal Amount? Brinda borrowed Rs.1000 to build a hut. She pays 5% Simple Interest. She lets the hut to Ramu and receives the rent of Rs.12.5 per month from Ramu. In how many Years Brinda ought to clear off the debt?

3. 4. 5.

6.

Pondicherry Engineering College

27

Training & Placement 7. The difference between the C.I. and the S.I. accrued on an amount of Rs.18000 in 2 Years was Rs. 405. Find the Rate of Interest. If the S.I. on a Sum of money at 5% p.a. for 3 Years is Rs.1500 Find the C.I. on the same Sum for the same period and at the same Rate? A man borrowed Rs.8000 at 12% per annum on S.I. He lent the whole Sum at 12% p.a on C.I. What will be his Gain after 2 Years?

8.

9.

10. Rs. 3757 is to be divided between A and B such that As Share at the end of 7 Years may be equal to Bs Share at the end of 9 Years. If rate percent be 10% p.a. Compound Interest. Find Bs Share 11. The Population of a certain village increases by 5% annually. Its present population is 8000. What will be the population after 3 Years?

1.11. Average
1.11.1. Concept: The Average or Mean or Arithmetic Mean of a number of Quantities of the same kind is equal to their Sum divided by the number of those Quantities. For example, the Average of 3, 9, 11, 15, 18, 19 and 23 is [3 + 9 + 11 + 15 + 18 + 19 + 23] / 7 = 98 / 7 = 14. 1.11.2. Formulae: 1) Average

Sum of Quantities Number of Quantities Average * Number of Quantities. Sum of Quantities Average

2) Sum of quantities 3) Number of quantities

= =

Problem 1: The Average of 9 Results is 30. If the Average of First Five Results is 48 and that of the last five is 53, Find the Fifth Result. Solution: Fifth Result = = = (48 * 5 + 53 * 5 30 * 9) (240 + 265 270) 235

Problem 2: A mans Average daily Expenditure is Rs.10 during May, Rs.14 during June and Rs.15 during July. Find the Average daily Expenditure for the Three months. Pondicherry Engineering College 28

Training & Placement Solution: As there are 31 Days in May, 30 Days in June and 31 in July. The total Expenditure = = = = = (10 * 31+14 * 30+15 * 31) rupees (310 + 420 + 465) Rs.1195 31 + 30 + 31 = 92 Rs.1195 / 92 = Rs.13 approx.

The number of Days The Average daily Expenditure 1.11.3. Problems to Solve:

1. The Mean of 100 Observations was calculated as 40. It was found later on that one of the Observations was misread as 83 instead of 53. Find the correct Mean. 2. The Average of 11 Observations is 60. If the Average of First Five Observations is 58 and that of the last Five is 56, then Find the Sixth Observation. 3. The Average Salary of a Staff of 18 Officers and 32 Clerks is Rs. 800. If the Average Salary of the Officers is Rs.1200, find the Average Salary of the Clerks. 4. A Cricketer has a certain Average for 9 Innings. In the Tenth Inning he scores 100 runs, thereby increasing his Average by 8 runs. Find his new Average. 5. Having scored 98 runs in the 19th Innings, a Cricketer increases his Average score by 4. What will be his Average score after the 19th Innings? 6. The Average of Ten numbers is 7. If each number is multiplied by 12 find the Average of new set of numbers. 7. The Average Temperature from Monday to Thursday is 48o and from Tuesday to Friday is 52o. If the Temperature on Monday is 42o, what was it on Friday? 8. The Average of marks obtained by 120 candidates was 35. If the Average of the passed candidates was 39 and that of the failed candidates was 15, find the number of those Candidates who passed the Examination 9. On an 800-mile trip, a car traveled half the distance at 80 miles per Hour and the other half at 100 miles per Hour. What was the Average Speed of the car? 10. The Average Height of 30 Boys out of a class of 50 is 160 cm. If the Average Height of the remaining Boys is 165 cm find the Average Height of the whole class (in cm).

Pondicherry Engineering College

29

Training & Placement

1.12. Permutations and Combinations


1.12.1. Concept: If an operation can be performed in m different ways; following which a second operation can be performed in n different ways, then the two operations in succession can be performed in n*m ways. If an operation can be performed in m different ways and another operation, which is independent of the first operation, can be performed in n different ways, then either of the two operations can be performed in (m + n) ways. Each of the different arrangements which can be made by taking some or all of given number of things or objects at a time is called Permutation. Let r and n be positive integers such that 1 < r < n. Then, the number of Permutations of n different things, taken r at a time, is denoted by the symbol nPr. Each of the different groups or selections which can be made by taking some or all of a number of things (irrespective of other) is called a Combination. The number of combinations of n different things taken r at a time is denoted by nCr. Factorial: The continued Product of first n natural numbers is called n! n! = 1*2*3*4* (n1) * n 0! = 1 1.12.2. Formulae: n 1. Pr 2. 3. Tips: If n is Even then the greatest value of nCr is nCn/2. If n is Odd, then the Greatest value of nCr is nCn+1/2 or nCn1/2. Problem 1: Find the number of Permutations and combinations which can be made by taking 6 items at a time from Eight given Distinct items without Repetition. Solution: The number of Permutations is 8P6 = = = n! / (n r)! 8! / 2! 20160 30
n

= n! / (nr)! = n! / [r! (nr)!] = nCn = 1 , nC1 = n

Cr

nCo

Pondicherry Engineering College

Training & Placement The number of Combinations is 8C6 = = = n! / r! (n r)! 8! / 6! 2! 28

Problem 2: How many different Permutations can be made out of the letters of the word, ASSISTANTS have taken all together? Solution: In this word, there are 10 letters composed of 4Ss, 2As, 2Ts, 1N, and 1I. Hence, the number of Permutations = 10! / 4! 2! 2! = 37800 Problems to Solve: 1 There are 15 buses running between Delhi and Mumbai. In how many ways can a man go to Mumbai and return by a different bus?

2. If there are 12 Persons in a party, and if each two of them shake hands with each other, how many handshakes happen in the party? 3. There are 8 students appearing in an examination of which 3 have to appear in a Mathematics paper and the remaining 5 in different subjects. In how many ways they are made to sit in a row if the candidates in Mathematics cannot sit next to each other? 4. In how many different ways can the letters of the word ALLAHABAD be permuted? 5. In how many different ways can the letters of the word TRAINER be arranged so that the vowels always come together? 6. How many words, each of 3 vowels and 2 consonants, can be formed from the letters of the word INVOLUTE? 7. How may different words can be formed with the letters of the word BHARAT. In How many of these B and H are never together? 8. A select group of 4 is to be performed from 8 men 6 women in such a way that the group must have atleast one woman. In how many different ways can it be done? 9. There are three different rings to be worn in four fingers with at the most one in each finger. In how many ways can this be done? 10. If there are 6 periods in each working day of a school, in how many ways can one arrange 5 subjects such that each subject is allowed at least one period?

Pondicherry Engineering College

31

Training & Placement

1.13. Percentage
1.13.1. Concept: The term Percent Means per Hundred or for every Hundred. It is the Abbreviation of the Latin phrase per centum. The term percent is sometimes abbreviated as p.c. The symbol % is often used for the term percent. A Fraction whose Denominator is 100 is called a Percentage and the Numerator of the Fraction is called rate percent, Example: 5% = 5 / 100 5 parts out of every hundred parts. Tips: 1. To convert a Fraction into a percent, multiply by 100 and put % sign. 2. If A is x% more than B, then B is less than A by [X / (100 + x) * 100] % 3. If A is x% less than B, then B is more than A by [X / (100 x) *100] % Problem 1: Find the Percentage Change in Area if the Length is increased by 20% and Breadth is decreased by 10% of a Rectangle. Solution: Increase in Length Decrease in Breath Percentage = = = = 20% 10% X + Y + (XY / 100) % 20 10 (20*10 / 100) = 8%

Problem 2: The Price of a Hindi book in 1987 was Rs. 100 but due to devaluation of the rupee it has risen to Rs. 250. What is the Percentage increase in its Price? Solution: Percentage Increase = = = [(Final Price Initial Price) / Initial Price]*100% [(250 100) / 100]*100% 150 %

1.13.2. Problems to Solve: 1. In an Examination 75 % of the candidates passed in English, 65% in Mathematics and 27% failed in both subjects. Find the Pass Percentage. A rise of 20% in the Price of rice compels a Person to buy 4 kg less for Rs. 80. Find the increased Price per kg.

2.

Pondicherry Engineering College

32

Training & Placement 3. A reduction of 20 percent in the Price of mangoes enables a man to buy 25 mangoes more for Rs. 40. Find the reduced Price of the basket containing 200 mangoes. A reduction of 12.5% in the Price of a dining table brought down its Price to Rs. 4375. Find the original Price (in Rs) of the table. A house owner was having his house painted. He was advised that he would require 25 kg of paint. Allowing for 15% wastage and assuming that the paint is available in 2 kg cans, what would be the cost of paint purchased, if one can costs Rs. 16? Swati spends 40% of her Salary on food, 25% on house rent, 15% on entertainment and 5% on conveyance. If her saving at the end of a month is Rs.1200, find her Salary per month. The Boys and Girls in a college are in the Ratio 3:2. If 20% of the Boys and 25% of the Girls are adults, find the percentage of students who are not adults. At an election a candidate who gets 35% of the votes is defeated by a majority of 150 votes. Find the total number of votes recorded. The petrol Prices are reduced by 10%. Find by how much a user must increase the consumption of petrol so as not to decrease his expenditure on petrol.

4.

5.

6.

7.

8.

9.

10. A candidate must get 33% marks to pass. He gets 220 marks and fails by 11 marks. What is the maximum number of marks?

1.14. Boats and Streams


1.14.1. Concept: If a boat moves against the stream that is in the Opposite Direction of the stream, it is called Upstream. If a boat moves along the stream that is in the direction of the stream, it is called Downstream. If Water of the river is moving, it is called a Stream. If the speed of the Water in the river is zero, it is called Still Water. 1.14.2. Formulae: If the speed of the boat be x kmph and the speed of the stream be y kmph then (a) Speed of the boat down stream (a) = ( X + Y) kmph (b) Speed of the boat upstream (b) = = = = ( X Y) kmph (Downstream speed + upstream speed) 2 (a + b). (a b).

(c) Speed of the boat in still Water

(d) Speed of the stream

Pondicherry Engineering College

33

Training & Placement Problem 1: A man can row 2 / 7th of a kilometer upstream in 25 minutes and return in 10minutes. Find the Speed of the man in still Water. Solution: Upstream speed Down stream speed Speed in still Water = = = Problem 2: A man can row upstream at 8 kmph and downstream at 10 kmph. Find the mans speed in still Water and the speed of current. Solution: Speed in still Water = (10 + 8) kmph = 9 kmph Speed of current = (10 8) kmph = 1 kmph. 1.14.3. Problems to Solve: 1. If a man can swim downstream at 8 kmph and upstream at 4 kmph, find the Speed in still Water. 2. A man can row upstream at 12 kmph and downstream at 17 kmph; find the Speed of the Stream. 3. A man can row a boat at 6 kmph in still Water. If the Speed of the Stream is 4 kmph, find the time taken to row a distance of 90 km down the Stream. 4. A man rows 13 km upstream in 5 Hours and 28 km downstream in 5 Hours. Find the Velocity of the Stream. 5. The Speed of a boat in still Water is 9 kmph and the Speed of the Stream is 1.5 kmph. A man rows to a place at a distance of 10.5 km and comes back to the starting point. Find the total time taken by him. 6. A man rows to a place 48 km far and comes back within 14 hours. He finds that he can row 4 km with the stream in the same time as 3 km against the stream. Find speed of the Stream. 7. A man rows upstream 12 km and downstream 28 km taking 5 hours each time. Find the velocity of Water. [(24 / 35) + (12 / 7)] / 2 [(84 / 35) x (1 / 2)] 1.2 kmph = = = [(2 / 7) / (25 / 60)] 24 / 35 kmph [(2 / 7) / (10 / 60)] =12 / 7 kmph

Pondicherry Engineering College

34

Training & Placement 8. A man who can swim 48 m/min in still Water swims 200 m against the current and 200 m with the current. If the difference between these two times is 10 min, find the speed of the current in m per min. 9. In a stream running at 2 km/hr, a motor boat goes 10 km upstream and back again to the starting point in 55 min. Find the speed of the motorboat in still Water. 10. A man can row 30 km upstream and 44 km downstream in 10 hr. Also, he can row 40 km upstream and 55 km downstream in 13 Hours. Find the rate of the current and speed of the man in still Water.

1.15. Time and Distance


1.15.1. Concept: The terms Time and Distance are related to the speed of a moving object. The Speed of an object is defined as the distance covered by it in a unit time of interval. It is obtained by dividing the distance covered by the object, by the time it takes to cover that distance. 1.15.2. Formulae: 1. Distance 2. Speed 3. Time Tips: 1. For converting kmph into m/sec multiply by 5/18 2. For converting m/sec to kmph multiply by 18/5. 3. If two Persons A and B start at the same time from two points P and Q towards each other and after crossing they take t1 and t2 Hours in reaching Q and P respectively then As speed Bs speed = t2 t1 = = = Speed * Time Distance Time Distance Speed

4. If a Person A covers a distance d1 km at s1 kmph and then d2 km at s2 kmph, then the Average speed during the whole journey is given by, Average speed = [s1*s2*(d1 + d2)] / [(s1*d2) + (s2*d1)] 5. If A goes from X to Y at s1 kmph. and comes back from Y to X at s2 kmph, then the Average speed during the whole journey is given by, Average Speed = 2s1*s2 / (s1 + s2)

Pondicherry Engineering College

35

Training & Placement 6. If a Person goes certain distance (A to B) at a speed of s1 kmph and returns back at a speed of s2 kmph. If he takes T Hours in all, then the distance between A and B is = T*[s1*s2] / (s1 + s2) Problem 1: In how many seconds does a 280 m long train moving at 208 kmph cross a platform of length, 250m? Solution: Distance covered Speed Time = = = = = 280 + 250 530m 208*5 / 18 = 57.78 m/sec Distance / Speed 530 / 57.78 = 9.17 Sec.

Problem 2: The distance between two cities A and B is 330 km. A train starts from A at 8 A.M. and travels towards B at 60 km / hr. Another train starts from B at 9 A.M. and travels towards A at 75 kmph. At what time do they meet? Solution: The Trains may meet x Hours after 8 a.m. Let the Trains be P and Q respectively Distance by P in x hrs + Distance by Q in (x 1) hrs 60x + 75 (x 1) x They meet at 11 A.M. Problem 3: Find the Length of the Bridge which a train 120 m long traveling at 54 kmph can cross in 30 seconds. Solution: Speed of the Train = 54 * 5 / 18 = Distance covered in 30 seconds = = Length of the Bridge = = = 1.15.3. Problems to Solve: 1. Vikas can cover a certain Distance in 1 hr 24 min by covering two third of the distance at 4 kmph and the rest at 5 kmph. What is the total distance? Pondicherry Engineering College 36 54 kmph 15 m/sec 15 *30 450m Distance covered Length of the Train 450 120 330m

= = =

330 330 3

Training & Placement 2. If a Boy walks from his house to school at the rate of 4 kmph, he reaches the school 10 min earlier than the schedule time. However, if he walks at the rate of 3 kmph, he reaches 10 min late. What is the distance of the school from his house? 3. I had to be at a certain place at a certain time and I found that I shall be 40 min too late, if I walk 3 km an hr and 30 min too soon, if I walk 4 km an hr. How far I have to walk? 4. If a man runs at 3 m per sec, How many kilometers does he run in 1 Hour 40 min? 5. A man travels uphill with an Average speed of 24 kmph and comes down with an Average speed of 36 kmph. The distance traveled in both the cases being the same, what is the Average speed during the journey? 6. A train running at the rate of 40 km an Hour meets a Person, walking along the line in the opposite direction at the rate of 4 km an Hour and passes him in 5 sec. Find the length of the train 7. A bullock cart has to cover a distance of 80 km in 10 Hours. If it covers half the distance in 3/5th of the time, What should be its speed to cover the remaining distance in the time left? 8. Excluding stoppages, the speed of the train is 45 kmph and including stoppages, it is 36 kmph. For how many minutes does the train stop per Hour? 9. A car starts with the initial speed of 40 kmph, with its speed increasing every Hour by 5 kmph. How many Hours will it take to cover a distance of 385 km? 10. Two Men start to walk together to a certain distance, one at 4 kmph and another at 3 kmph. The former arrives half an Hour before the latter. Find the distance.

1.16. Problems on Ages


1.16.1. Concept: To solve these Problems, the knowledge of linear equations is essential. There may be three situations: 1. Age some Years ago. 2. Present age. 3. Age some Years hence. Simple linear equations are to be framed and the solution can be obtained.

Problem 1: The Ages of two Persons differ by 10 Years. If 5 Years ago, the elder one was 2 times as old as the younger one, Find their Present Ages. Pondicherry Engineering College 37

Training & Placement Solution: Let the age of the Younger Person be x Years. Then, age of the Elder Person = (x + 10) Years 2(x5) = (x + 10 5) 2x 10 = x+5 x = 15 Hence, their Present Ages are 15 Years and 25 Years. Problem 2: The Present Age of a mother is 4 Years more than Four Times the age of her Daughter. Four Years hence, mothers age will be 6 Years more than twice the age of the Daughter. Find the Present Age of the mother? Solution: Let the Daughters present age be x Years. Then, Mothers Present Age = (4x + 4) Years Four years hence (4x + 4 + 4) = 2(x + 4) + 6 4x + 8 = 2x + 14 x = 3. Hence, Mothers Present Age = (4x + 4) = (4*3 + 4) = 16 Years. 1.16.2. Problems to Solve: 1. One Year ago, a father was four times as old as his son. In 6 Years time his age will exceed twice his sons age by 9 Years. Find the Ratio of their present ages 2. The Average age of five members of a family is 21 Years. If the age of the grandfather is included, the Average is increased by 9 Years. Find the age of the grandfather 3. The difference between the ages of two Persons is 10 Years. 15 Years ago, the elder one was twice as old as the younger, Find the present age of elder 4. The ages of Ram and Shyam differ by 16 years. Six years ago, Mohans age was thrice as that of Rams. Find their present age? 5. One Year ago, Promila was four times as old as her daughter. Six Years hence, Promilas age will exceed her daughters age by 9 Years. Find the Ratio of the present ages of Promila and her daughter 6. The Ratio of Laxmis age to the age of her mother is 3:11. The difference of their ages is 24 years, find the Ratio of their ages after 3 years? 7. Three years ago, Xs age was double of Ys. Seven years hence the Sum of their united ages will be 83 years. find the age of X today

Pondicherry Engineering College

38

Training & Placement 8. The Ratio of fathers ages to sons age is 3:1. The Product of their ages is 243. What will the Ratio of their ages be 3 years hence? 9. Narmathas father is now four times her age. In 5 years, he will be 3 times her age. In How many years, will he be twice her age? 10. Three Years ago, the Average age of a family of 5 members was 17 Years. A baby having been born, the Average of the family is the same today. What is the age of the baby?

1.17. Profit and Loss


1.17.1. Concept: The aim of every Business is to earn Profit. The terms that describe Profit is Cost Price and Selling Price. The Cost Price of an article is the Price at which an article has been purchased. It is abbreviated as C.P. The Selling Price of an article of an article is the Price at which an article has been sold. It is abbreviated as S.P. If the selling Price of an article is more than the cost Price, there is a Gain or Profit. If the cost Price of an article is greater than the selling Price, it is defined as Loss. 1.17.2. Formulae: 1. 2. 3. 4. 5. 6. Profit (P) Loss (L) = = S.P. C.P C.P. S.P (P /C.P)* 100 % (L /C.P)* 100 % [100 + P%] * C.P. 100 [ 100 ]* S.P. 100 + P% [(100 Loss %) /100] * C.P. [100 / 100 Loss%] * S.P.

Profit %( P %) = Loss % S.P. C.P. = = =

7. 8. Problem 1:

S.P. C.P.

= =

A Girl buys an article for Rs. 17.50 and sells it for Rs. 18.60. Find her Gain percent. Solution: C.P = Rs.17.50 S.P = Rs.18.60 So, Gain = Rs. (18.60 17.50) = Rs.1.10 Gain% = ((1.10 / 17.50)*100) % = 6.28% Pondicherry Engineering College 39

Training & Placement Problem 2: An Article is bought for Rs. 80 and sold at a Gain of 20%. What is Selling Price of the book? Solution: Gain = Rs. 5, so Gain % 1.17.3. Problems to Solve: 1. A Man bought 5 apples for Rs. 3 and sold each for Rs. 2. What did he Gain or Loss? 2. A man sold two pens at Rs.20 each. He sold one at a loss of 10% and the other at a Gain of 10%. Find his Goss or Gain %. 3. If Cost Price of 8 articles is equal to the selling Price of 10 articles. Find the Gain or Loss. 4. Vikas bought paper sheets for Rs. 7200 and spent Rs. 200 on transport. Paying Rs. 600, he made 330 boxes, which he sold at Rs. 28 each. Find his Profit percentage. 5. A sells a bicycle to B at a Profit of 20% and B sells it to C at a Profit of 25%. If C pays Rs.1500, What did A pay for it? 6. The profit earned after selling an article for Rs.625 is the same as loss incurred after selling the article for Rs. 435. What is the Loss percent? 7. A machine is sold at a Profit of 10%. Had it been sold for Rs. 80 less, there would have been a loss of 10%. Find the C.P of the machine. 8. A cloth merchant says that due to slump in the market, he sells the cloth at 10% loss but he uses a false metre scale and actually Gains 15%. Find the actual length of the scale. 9. If a Person makes a Profit of 10% on 1/4th of the quantity sold and a loss of 20% on the rest, then what is his Average percent Profit or Loss? 10. When a producer allows 36% commission on retail Price of his Product, he earns a Profit of 8.8%. What would be his Profit percent, if commission is reduced to 24%? = (5/20) * 100 =25%

1.18. Calendar
1.18.1. Concept: The Process of Finding the day of a given date depends upon the number of Odd Days present. The Days more than the complete number of weeks in a given period is called Odd Days. The Year (except century) which is divisible by 4 is called a Leap Year, where as century is a leap Year by itself when it is divisible by 400. Leap Year has 366 Days, i.e. 52 Weeks and 2 Odd Days. An Ordinary Year has 365 Days, i.e. 52 weeks and 1 Odd day. Pondicherry Engineering College 40

Training & Placement Tips: 1. A Century has 76 Ordinary Years and 24 Leap Years i.e. 76 + 48 Odd Days = 124 = 17 weeks and 5 Odd Days. 200 Years contain 3 Odd Days. 400 Years contain 1 Odd day. Since the order is Cyclic (i.e. every Century has 0, 1, 3 or 5 Odd Days) The First Day of a Century must either be Monday, Tuesday, Thursday or Saturday. Last Day of a Century cannot either be Tuesday, Thursday or Saturday. To find the Day of the Week on a particular date when reference date given: Find the net number of Odd Days for the period between the reference date and the given date. To find the Day of the Week on a Particular Date when no Reference Day is given: Count the net number of Odd Days on the given date and write the day corresponding to the Odd day.

2. 3. 4. 5. 6. 7.

8.

Odd Days: Sunday Monday Tuesday Wednesday Thursday Friday Saturday

=0 =1 =2 =3 =4 =5 =6

Month code: Ordinary Year has the Odd Days as follows January = 0 Odd Days February March =3 April May =1 June June =6 August September =5 October November =3 December

=3 =6 =4 =2 =0 =5

Month code for leap Year adds 1 with ordinary Years corresponding month code. Problem 1: 11th January 1997 was a Sunday. What day of the Week was on 7th January 2000?

Pondicherry Engineering College

41

Training & Placement Solution: Total number of Days between 11th January 1997 and 7th January 2000 = (365 11 Days) in 1997 + (2*365 Days) in 1998 and 1999 + 7 Days in 2000. = (50weeks + 4 Odd Days) + 2*(52 weeks and 1 Odd day) + 0 Odd day = 6 Odd Days Hence, 7th January 2000 would be 6 Days ahead of Sunday, i.e. it was on Sunday. Problem 2: What Day of the Week was 5th June 1999? Solution: (A+B+C+D)-2, Take the Remainder 7 A = Divide the year by 7 and take the Remainder 1985/7, Remainder is 4 B = Divide the year by 4. Take the Quotient and divide it by 7. Take the Remainder. 1985/4 = 496; 496/7, Remainder is 6 C = Divide the Date by 7 and Take the Remainder. 2/7, Remainder is 2 D = Month Code June Code is 4 (4+6+2+4)-2 = (14-2)/7 = 5 7 th Hence, 5 June 1999 was Saturday. 1.18.2. Problems to Solve: 1. 2 nd July 1985 was Wednesday. What Day of the Week was 2nd July 1984? 2. India got Independence on 15th August 1947. What was the Day of the Week? 3. The First Republic Day of India was celebrated on 26th January 1950 What was the Day of the Week on that Date? 4. Smt. Indira Gandhi died on 31st October 1984.What was the Day of the Week? 5. What was the Day of the Week on 26th June, 2002?

Pondicherry Engineering College

42

Training & Placement

1.19. Clocks
1.19.1. Concept: The Circumference of a Dial of a Clock is divided into 60 Equal Parts called Minute Spaces. The Clock has two hands the Hour Hand and the Minute Hand. The Hour hand (or short hand) indicates time in Hours and the Minute Hand (or the long hand) indicates time in Minutes. In an Hour, the Hour hand covers 5 minute spaces while the minute hand covers 60 minute spaces. Thus in One Hour or 60 minutes, the minute hand gains 55 minute spaces over the Hour hand. In every Hour the hands are twice at Right angles. In every Hour both the hands coincide once. 1.19.2. Formulae: To find the Angle between the hands = 30 h 11m or 11m 30 h, where h denotes Hour and m denotes Minutes. 2 2 Tips: 1. The Minute Hand moves 12 times as fast as the Hour Hand. 2. The Two Hands of the Clock will be together between H and (H + 1) o clock at (60H / 11) minutes past H oclock. 3. The Two Hands of the Clock will be at Right Angles between H and (H + 1) o Clock at (5H 15)12 / 11 Minutes past H o Clock.

4. If the Minute Hand of a Clock overtakes the Hour Hand at intervals of M minutes of correct time. The Clock gains or loses in a Day by [(720 / 11) M]*(60*24 / M) minutes. Problem 1: Find the Angle between the Minute Hand and Hour Hand of a Clock at 07:20. Solution: Angle between the two hands at 7: 20 = 30h (11m / 2) = 30x7 (11*20 / 2) = 100 Degrees. Problem 2: How many times do the Hands of a Clock Coincide in a Day? Solution: The Hands of a Clock Coincide 11 times in every 12 Hours The Hands Coincide 22 times in a Day. Pondicherry Engineering College 43

Training & Placement 1.19.3. Problems to Solve: 1. Find the Angle between the Hour Hand and the Minute Hand of a Clock when the time is 3.25? At what Time between 4 and 5 oclock will the Hands of a Watch Point in Opposite Directions? At what Time between 9 and 10 oclock will the hands of a Watch be together? At what time between 5.30 and 6 oclock will the hands of a Watch be 90 ? How much does a Watch Lose per day, if its hands coincide every 64 Minutes?

2.

3. 4.

5.

2. Analytical Reasoning
Concept: As it is very much evident from the word analytical itself that this type of reasoning is based on the analysis of the statements which are there in the question, the approach for solving analytical reasoning questions varies from question to question as analytical reasoning questions are of different types. There are two ways of solving analytical reasoning questions. The first way is the one which consists of the general method for solving any type of analytical reasoning questions. This method consists of a standard approach which must be followed in all the analytical reasoning questions (based on the need).The other approach is based on the specific type of the questions. Tips for Solving Analytical Reasoning questions: 1. Read the statements very thoroughly and repeatedly to clearly understand the meaning (known or unknown) which they carry. 2. Do not jump from one statement to the other before completely understanding the statements. 3. Use notations, symbols and abbreviations wherever required, preferably. 4. Mark the keywords which are present in the statements. 5. Organize the information given in the question in the form of suitable Tables, Maps, or Tree. 6. Do not make unnecessary assumptions.

2.1. Deduction
2.1.1 Concept: Deduction Means reducing two statements into one or three statements into one. It is also known as Syllogism. Syllogism is a noun which means a form of reasoning in which a conclusion is drawn from two statements, i.e., Deductive Reasoning. In more clear terms Syllogism is a mediate deductive inference in which two propositions are given in such an order that they jointly or collectively imply the third. Thus Syllogism can be defined Pondicherry Engineering College 44

Training & Placement as a form of reasoning in which the conclusion establishes a relation between two terms on the basis of both terms being related to the same third term as derived in the premises. For example, 1. All human beings are mortal. 2. The child is a human being. 3. The child is mortal. The conclusion is reached through the medium of a middle term, i.e., human being, with both subject (children) and the predicate (mortal). Therefore, in a Syllogism two premises are necessary to arrive at a conclusion. Types: The types of deduction are: Deduction in two statements and Deduction in three statements. If the statement starts with, All, No Universal Statement Some, Many Particular Statement No, Not Negative Statement Universal Affirmative: If the statement starts with ALL or NO it is Universal Affirmative. Universal Negative: If the statement starts with NO it is Universal Negative. If the statement starts with ALL and if the statement has the word Not it is Universal Negative. Particular Affirmative: If the statement starts with SOME or MANY it is Particular Affirmative. Particular Negative: If the statement starts with SOME or MANY and if the statement has the word Not it is Particular Negative. DISTRIBUTED Subject Universal Affirmative Universal Negative Particular Affirmative Particular Negative Rules for Deductions: The middle term must be distributed at least once. Two statements should have only Three Distinct terms. Pondicherry Engineering College 45 X X X UNDISTRIBUTED Predicate X X

Training & Placement The Middle term should not come in the conclusion If both the statements are Particular there would be No Conclusion If both the statements are Negative there would be No Conclusion. If one Statement is Particular, the Conclusion would be Particular If one Statement is Negative, the Conclusion would be Negative. All All All No No No Conclu. Some Many No Conclu. One Conclusion. Two Conclusions. Two Conclusions. One Conclusion. All Some Some All No No Some No Some, Not

Stat. 1 Stat. 2 Conclusion

Universal Affirmative Universal Negative Particular Affirmative Particular Negative Problem 1:

All pens are pots. All pots are jacks Solution: Statement 1: Universal affirmative Subject (pens) Distributed Predicate (pots) Undistributed Statement 2: Universal affirmative Subject (pots) Distributed Predicate (jacks) Undistributed Conclusion: All pens are jacks 2.1.2 Problems to Solve 1. All bats are keys All locks are keys No keys are doors 2. All horses are fruits Some boxes are trees Some trees are horses 3. All dogs are Parrots No train is dog Some buses are trains

Pondicherry Engineering College

46

Training & Placement 4. Some guns are Pistols All Pistols are Bombs All Crackers are Pistols 5. Some fools are professionals All engineers are fools 6. Some dogs are cats None of the cats are cows 7. Shyam is bachelor All bachelors are intelligent 8. All sports are enthusiastic Cricket is not a sport 9. No magazines are caps All caps are cameras 10. All poles are Guns Some Boats are not Poles

2.2 Linear Sequencing


2.2.1. Concept: Arranging the items in a linear fashion is called Linear Sequencing, i.e. arranging it in single row manner. There may be 45 questions asked from a single linear pattern given. The direct condition should be found first and then the question can be solved, because the remaining conditions are based on this direct condition. Problem 1: There are five Persons (A, B, C, D, and E) in a room who are seated in a single row in the following manner: A does not sit immediately next to E. E and D sit immediately next to each other. C is at the extreme right end of the row. A is immediately to the left of C. Who must sit in the middle seat of the row? Solution: First draw linear arrangement as given below; C is at the extreme right. i.e. __ __ __ __ C A is to the immediate left of C. i.e. __ __ __ A C E does not sit next to A, So occupy 1 or 2 seat i.e. __ E __ A C or E __ __ A C D sits next to E. E takes 2 seats: i.e. either DEBAC or BEDAC E takes 1 seat: EDBAC Hence Middle Seat: B or D

Pondicherry Engineering College

47

Training & Placement

2.2.2. Problems to Solve: 1. Eleven Students A, B, C, D, E, F, G, H, I, J and K are sitting in the first row of the class facing the teacher. D is to the immediate left of F, who is second to the right of C. A is second to the right of E, who is at one end. J is the immediate neighbor of A and B is third to the left of G. H is to the immediate left of D and third to the right of I. I. Who is sitting in the middle of the row? II. Which of the following groups of friends are sitting to the right of G? a. CHDE b. CHDF c. IBJA d. ICHD 2. In a Car Exhibition, seven cars of seven different companies such as Cadillac, Ambassador, Fiat, Maruthi, Mercedes, Bedford and Fargo were displayed in a row, facing the east such that: Cadillac car was to the immediate right of Fargo. Fargo was fourth to the right of Fiat. Maruthi car was between Ambassador and Bedford. Fiat, which was third to the left of Ambassador Car, was at one end. I. Which of the following was the correct position for the Mercedes? a) Immediate right of Cardilac b) Immediate left of Bedford c) Between Bedford and Fargo d) Fourth to the right of Maruthi II. Which cars are on the either side of the Cardilac cars? a) Ambassador and Maruthi b) Maruthi and Fiat c) Fiat and Mercedes d) none of these 3. There are seven books one each on Psychology, Hindi, English, Sociology, Economics, Education and Accountancy, lying on the table one above the other. Sociology is on top of all the books. Accountancy is immediately below Education which is immediately below Sociology. Economics is immediately above Psychology but not in the middle. Hindi is immediately below psychology. I. Economics is inbetween which of the following books? a) Accountancy and Education c) English and Psychology b) Psychology and Hindi d) Psychology and Sociology

II. If Sociology and English, Accountancy and Hindi and Education and Psychology interchange their positions, which book will be between Psychology and Sociology? a) Accountancy b) Psychology c) Hindi d) Economics 4. In a pile of reading material, there are novels, story books, dramas and comics. Every novel has a drama next to it, every storybook has a comic next to it and there Is no storybook next to a novel. If there be a novel at the top and the number of Books is 40, what would be the order of the books in the pile? Pondicherry Engineering College 48

Training & Placement a) NSCD b) NDSC c) CSDN d) DNCS

5. There are 5 houses H1, H2, H3, H4, H5 in a row H1 is to the right of H2 & H5 is to the left of H3 and right of H1, H2 is to the right of H4. Which is to the house in the middle? 6. The buses stand in a row. One of the 2 buses at the extreme end is red and the other blue. A yellow bus is standing to the right of green. An orange bus is to the left of blue bus & Green bus stands between the red and yellow buses. Orange bus stands at ________ Position from right. Which bus stands at the centre? Yellow bus stands to the left of _____? 7. 5 people A, B, C, D, and E read a novel. One who finishes first gives it to C & one who reads last had taken it from A. E has not read it first or lastly two people read between B & A. Whom did B pass the novel to? Who read the novel last? 8. There are 6 Persons (A, B, C, D, E & F) in a Team who are seated in a linear manner. A sits in between E and D. D sits in one extreme of the row. There are two places in between B and E. How many arrangements are possible? If C takes the 2nd spot and E shifts the place to B How many people are sitting in between E and D? 9. M, N, O, P, Q, R & S are 7 Persons who are sitting in a row. O sits to the immediate left of S, and R is to the immediate right of P. N has equal number of people on either side of him. M is to the immediate left of N and there are exactly three Persons between M and S. If the seven Persons sit facing forward, viewing from front, then who sits to the immediate right of Q? If Q wants to sit to the immediate left of N, then with whom should he exchange the seat? How many Persons are there between R and Q? 10. Nine Persons A, B, C, D, E, F, G, H, I are seated in a row. The seats are numbered from 19 from left to right as per the following restrictions. C, F, I sit together. E is to the immediate right of H. B sits in the fourth seat from the left end and D sits in the third seat from right end. E is not between B and D. If G sits to the immediate left of D then who will sit in the centre seat of the row? If F sits between C and I, then in how many ways altogether can all of them be seated in the row? Which of the following is at one end of the row? 11. Five Persons L, M, N, O, and P are sitting in a line. L is to the right of N; M is to the left of P. N is to the right of O; M is to the right of L. Who is sitting second from your right side? Pondicherry Engineering College 49

Training & Placement Identify the Persons sitting on the extremities.

2.3. Double Lineup


2.3.1. Concept: The question will consist of at least two different variables. We have to form a table. There will be more than one question per passage. Problem 1: P, Q, R, S, T, U, V, W are 8 employees allocated with eight different lockers numbered 18. The lockers are arranged in four rows with two lockers in each row. Lockers 1 and 2 in top row from left to right and lockers 7 and 8 in bottom row from left to right. Lockers 3 and 4 are in the second row from top arranged from right to left and so do 5 and 6. P has been given locker 1 while v has been allocated locker 8. T has locker just above that of Q which is just above that of R whereas W locker is in bottom row. 1. Draw the table? 2. Which locker is allocated to employee Q? 3. How many arrangements are possible? 4. If W and P are interchanged what does the table look like? 5. If S is given locker 4 how many solutions are possible? Solution: 4 Rows and 4 Columns are given. Fitting the data in the table results in Left 1 4 6 7 Right 2 3 5 8

P S/U S/U W 1) 2) 3) 4)

T Q R V

Employee Q is allocated locker 3 There are 2 arrangements possible W would take the top row and P would take the bottom row If S is given locker 4 there is only 1 solution possible.

2.3.2. Problems to Solve 1) On the shelf are placed six volumes side by side labeled A, B, C, D, E and F. 3 volumes C, B and E have blue covers while the rest have red covers. D and F are new volumes while the rest are old volumes. A, C and D are law reports while the rest are gazetteers Which is the red colored new law report volume? Which two old gazetteers have blue covers?

Pondicherry Engineering College

50

Training & Placement 2) P,Q,R,S,T and U are 6 post offices. In each of which one is a post man out of A, B, C, D, E and F workers. No postman works in two post offices. A works neither in post office P nor in S, while B works neither in post office T nor in U. C works neither in post office Q nor in R but D works in T and A works in Q. If B works in post office P and C does not work in post office U then E works in (a) S (b) U (c) R (d) R or U 3) 4 people (A, B, C & D) sit and have dinner in a room. The various dishes provided are (W, X, Y & Z). Each of them has one dish each as follows, If A has Y, B cannot have W, C has Z. What are the different arrangements possible? 4) There were 6 friends A, B, C, D, E & F. Every one is good in one of the following Games, Baseball, Volleyball, Carrom, Hockey, TT and Polo. Each owns a different colored car Yellow, Green, Black, White, Blue and Red. D plays Polo and has yellow car. C does not play TT & Hockey and owns neither blue nor yellow car. E has white car and plays Baseball. B does not play TT and has red car. A plays Carrom and he has black car. Who plays Volley ball? Which colored car F owns? 5) 6 friends Ramesh, Dinesh, Lokesh, Nilesh, Shaliesh & Hitesh work in different companies namely P, Q, R, S, T & U and each one wears company sponsored different coloured tie such as Blue, Green, Pink, Yellow, Purple & Red though not necessarily in the same order. The one wearing Blue tie works in Company S and the one wearing Green tie works in Company P. Hitesh does not work in company R. Ramesh wears Pink tie and works in Company Q. Nilesh does not work in Company T and purple colour tie is not sponsored by Company R. Shaliesh works in company U and neither Nilesh nor Dinesh works in company S. Company T does not Sponsor Purple or Yellow colored tie and Lokesh works in Company P. Which Colour Tie is sponsored by company R? Which of the following Colour Company Person combination is correct? a) Green R Nilesh b) Blue S Lokesh c) Red T Dinesh d) Yellow R Shaliesh Which of the following is true? a) Company U sponsored Green tie c) Nilesh works in T

b) Shailesh wears Red tie d) Red Colour is sponsored by T

6) Among Ram, Lakhan and Kishan are a doctor, a teacher and an engineer. They are married to Radha, Sita and Gita not in that order. Each of their wives is working. The day Ram and Lakhan saw the film Sita aur Gita, Sita and Gita saw the film Ram Lakhan. Kishans wife is an artist and regularly holds exhibitions at Venkatappa art gallery, Bangalore. Ram and Gita are having a love affair secretly. Teachers wife teaches in St. Josephs Convent. Radha is married to the Engineer.

Pondicherry Engineering College

51

Training & Placement Sita is a good cook and one can find her recipes in every issue of Womens Era magazine. Who is the Doctor? a) Ram b) Lakhan c) Kishan d) Cant Say Who is Rams wife? a) Gita b) Sita c) Radha d) Data insufficient Who is the Artist? a) Gita b) Sita c) Radha d) Cant Say Who is Radhas Husband? a) Ram b) Lakhan c) Kishan d) Cant Say Who is married to the teacher? a) Radha b) Sita c) Gita d) Cant Say Who is the Engineer? a) Ram b) Lakhan c) Kishan d) Data Insufficient 7. I like the following 5 Places in India A, B, C, D, E because of the following Characteristics. B and E have Seashores. B, D & A are metropolitan. E is a peaceful city. A & E are is of historic significance. Which city has seashore and historic significance? Which city is both a metropolitan and has seashore? Which city is not peaceful but has historic significance? 8. Six Products U, V, W, X, Y and Z are display windows of a shop. There are six display windows number 1, 2,3,4,5 and 6. One Product is to be put in one window. Moreover U cannot be immediately to the left or immediately to the right of V. W must be immediately to the left of X, Z cannot be in window number 6. If X is placed in window no.3, W must be placed in which window? If U is placed in window no.3, immediately to the right of X which Product must be placed in window no.5? There are four friends Dinesh, Sunil, Rahul and Atul. One of them studies Commerce and plays Golf and Table tennis, Dinesh and Sunil study Maths. Dinesh plays cricket. Both the Maths students play Football. Rahul is a student of Physics. The Physics student plays Football and Badminton. All the friends play two games each and study one subject each. Who is the student of commerce? Who does not play Football? Who plays cricket? Who studies Maths and plays Cricket?

9.

10. Five executives of CHOGM hold a conference in Delhi. Mr. A can speak Spanish and Hindi Mr. B understands Spanish and English Mr.C converses in English and Hindi. Mr. D speaks French and understands Spanish quite well. Mr. E, a native Indian, can also speak French. Which of the following can act as an interpreter when Mr. C and Mr. D wish to converse ? a) Only Mr. A b) Only Mr. B c) Only Mr. E d) Mr. A or Mr. B e) any of the other three executives. Pondicherry Engineering College

52

Training & Placement Which of them can not converse without an interpreter? a) Mr. B and Mr. E b) Mr. A and Mr. B c) Mr. A and Mr. C d) Mr. B and Mr. D e) Mr. A and Mr. E Of the languages spoken at this conference, choose the least common language: a) English and Spanish b) English and French c) Hindi and Spanish d) English and Tamil e) French and Spanish

2.4. Ordering
2.4.1. Concept: Items are to be arranged based on the conditions given. The questions will be based on the Persons weight, height, age, rank etc. Here the terms like greater than, less than, not equal to will come into play. Types: Ordering: If More than one Variable is given in the problem, use ordering method according to the condition. Elimination: If the Problem contains only one variable, then use elimination or ordering method, according to the condition. Problem: K, L, M, N, O and P are 6 Men. Neither M nor N is the tallest. L is taller than P but shorter than K. O is as tall as P but taller than M. Which is the tallest? Solution: The Persons are K, L, M, N, O and P. Statement 1: Neither M nor N are tallest So, Remaining: K L O P Statement 2: L is taller than P So, Remaining: K L O Statement 3: L is shorter than K So, Remaining: K O Statement 4: O is as tall as P. Since P Eliminated, O is not. Hence: K is the tallest.

Pondicherry Engineering College

53

Training & Placement 2.4.2. Problems to Solve 1. Four People Arti, Banti, Chetan and Dolly have Rs. 100 with them. Arti and Banti together have as much money as Chetan and Dolly. Arti has more money than Banti. Chetan has half as much money as Dolly. Arti has Rs. 5 more than Dolly. How much does Banti have? a) 12.03 b) 11.6 c) 13.3 d) None of these Who has the second highest amount? a) Arti b) Chetan c) Dolly d) Banti 2. A, B, C, D, E, and F are six students in a class. B and C are shorter than F but heavier than A. D is heavier than B and taller than C. E is shorter than D but taller than F. F is heavier than D. A is shorter than E but taller than F. Who among them is the Tallest? Who is the third tallest from the top when arranged in descending order? Who among them is the lightest? 3. I have 5 balls B1, B2, B3, B4 & B5 all of different weights. B1 is twice as much as B2, B2 is 4 times heavier than B3, B4 is twice of B3, but of B5s weight. B5 is less than B1 but more than B3 in weight. Find which is lightest. 4. A, B, C, D and E are five friends. B is elder to E, but not as tall as C. C is younger to A, and is taller to D and E. A is taller to D, but younger to E. D is elder to A but is shorter in the Group. Which of the following pair is elder to D? a) BA b) BC c) BE d) EA e) None of these If another friend F is taller than C how many of them will be between F and E according to height? a) None b) one c) two d) three e) None of these 5. J, K, L, M, and N are five Boys in a class. They are ranked in the order of their heights from tallest to shortest and in the order of cleverness from cleverest to dullest. K is taller than N, but not as clever as J and L, whereas M is the cleverest of all but shorter than J. While L is shorter than M but taller than K, L is not as clever as J. No two Persons got the same ranks in any parameters. Who is the third tallest? How many people are shorter than K? If L is the third cleverest then who is the dullest? 6. H is six months younger to Sita while B is three months younger to A, when Sheila was born, B was 2 months old. A belongs to the age group of five while H belongs to the age group of six. Who among them is the oldest? 7. Raksha is younger than Saksha and older than Sita. Anju is younger than Manju and older than Sanju. Manju is younger than Sita and older than Jugal. Jugal is younger than Sanju and older than Mughal. Saksha is younger than Beena and older than Heena. Heena is older than Raksha. Pondicherry Engineering College 54

Training & Placement Who is the Youngest? a) Anju b) Sanju c) Jugal d) Mughal

Who precedes Sanju, Jugal, Mughal in age? a) Manju b) Sita c) Anju d) Raksha 8. There are five friends Sachin, Kunal, Mohit, Anuj and Rohan. Sachin is Shorter than Kunal but taller than Rohan. Mohit is the tallest. Anuj is little shorter than Kunal and little taller than Sachin? Who is the Shortest? Who is the second tallest? 9. A Blacksmith has five iron articles A, B, C, D and E, each having a different weight. A weighs twice as much as B B weighs four and a half times as much as C C weighs half as much as D D weighs half as much as E E weighs less than A but more than C Which is the lightest in weight? Arrange the articles in descending order of their weights. 10. Among 5 friends, Mohan is older than Raju but not as old as Lalit. Lalit is Older than Neelesh and Kabir. Neelesh is younger than Raju but not the youngest. Who is the fourth in the descending order of age?

2.5. Seating Arrangement


2.5.1. Concept: It is the manner in which the people are seated in Place. Based on the question, draw the shape like Square, Triangle, etc. Mostly it is in the form of a circle. We have to draw the Shape which is given in the question and arrange them according to the conditions given. Problem 1: P, Q, R, S and T are 5 people who sit at a round table. P sits two tables to the left of R and Q sits two tables to the right of R. 1. If S is in between Q and R what is the arrangement? 2. How many different arrangements are possible? 3. If S is not sitting between Q and R then who is sitting between Q and S? Solution: 1. If S is between Q and R then the arrangement is QSRTP. 2.Two different arrangements are possible. 3.Either P is sitting when viewed anticlockwise or T and R when viewed clockwise. Pondicherry Engineering College 55

Training & Placement

2.5.2. Problems to Solve: 1. There are five different houses A to E in a row. A is to the right of B and E is to left of C and right of A. B is to the right of D. Which of the houses is in the middle? a) A b) B c) D e) E 2. P, Q, R, S, T and U are seated in a circle with P opposite to S. T has two seats in between himself and U. U is to the immediate left of P. What is the formation like? How many possibilities are there? 3. If 6 people A, B, C, D, E and F sit at a round table with the following conditions given how does the arrangement look like: A sits opposite to E. D sits to the immediate left of A and B sits two places to the right of A.

4. Eight friends A, B, C, D, E, F, G and H are sitting in a circle facing the centre. B is sitting between G and D. H is third to the left of B and second to the right of A. C is sitting between A & G and B & E are not sitting opposite to each other. Who is third to the left of D? 5. A group of eight members sit in a circle. D is between A and F and is opposite to G. E is to the right of A but on the left of C, whose right hand neighbor is G. B enjoys having H to his left and F to his right. Find the member who is diagonally opposite to A. 6. Six friends A, B, C, D, E and F are seated in a closed circle facing the centre. E is to the left of D. C is between A and B. F is between E and A. Who is to the left of B? 7. Six Persons A, B, C, D, E and F are standing in a circle. B is between F and C; A is between E and D; F is to the left of D. Who is between A and F? 8. L, M, N, P, Q, R and S are sitting in a circle and playing cards. N who is the neighbor of P, is not the neighbor of R. Q is second to the left of R. N is second to the left of S , who is the neighbor of M. Who is immediate right of S? 9. 6 people A, B, C, D, E and F sit around a table for dinner. Since A does not like C, he doesn't sit either opposite or beside C. B and F always like to sit opposite each other. If D is adjacent to F then who is adjacent to C? (a) E and B (b) D and A (c) D and B (d) none

10. P, Q, R, S, T, V and W are sitting around a circle facing the center. R is third to the right of V who is second to the right of P. T is the second to the left of Q who is second to the left of W. V is sitting between S and W. Who is third to the left of T?

Pondicherry Engineering College

56

Training & Placement

2.6. Selections
2.6.1. Concept: Selecting a part out of whole. Infer from the statements without Assumptions. Select the item from the whole according to the condition and start to solve from the direct condition. Write the condition in a shortcut like, A and B together Means A = B If A come and B will not come Means A B If B is selected, A and C will not be selecteed which Means B A, C Problem: Three students J, B, S have to select 3 subjects out of 6 subjects P, Y, M, C, H, and L. If L is selected then P cannot be selected. If S selects C then J does not select L. If B selects Y then S cannot select C and if J selects P then S selects H and L. If S selects L and B selects Y then S cannot select? Solution: S selects L; B selects Y; L is selected P cannot be selected. (given) P cant be selected If B selects Y then S cannot select C. (given) So C cant be selected Answer: P and C 2.6.2. Problems to Solve: 1. Four Students are to be selected from T, U, V, W, X, Y and Z. If T is selected then Z cannot be there. Y cannot come with W. U should be present in the team. Y and X come together. V cannot be a part of the team. Find the number of possibilities If T comes in the team who are its members? 2. In a class of 7 members they are going to select 4 members 2 from Boys and 2 from Girls. The Boys are A, B, C, D and Girls are X, Y, Z. Y & C can be together A& X are always together C & D cannot be together but C & B are always together. If A is selected who are the remaining members? If B is selected who are the remaining members? Who is the Person who will be always there in the team? 3. A team of 5 is to be selected for a Basketball tournament. The good players among, Boys are Amit, Bhel, Chaitanya, Dilip & Imran, Among Girls are Priti, Rani, Kiran and Sonia. The selection criteria are such that Amit and Sonia have to be together. Pondicherry Engineering College 57

Training & Placement Priti however cannot be with Rani, similarly we cannot have Rani with Bhel or Dilip with Kiran, and however Chaithanya and Imran have to be together. If two of the members have to be Boys, the team who will be the members of the team? Now if one of the members is Rani, who would be the others? Now let two members be Girls and Dilip is one of the members too, then the who would be the members of the team ? 4. City high school must put together a debating team consisting of four debaters. There are candidates of equal ability X, Y & Z who are seniors and A, B, C, D who are juniors. The school requires that they should be two seniors and two seniors in the team. It is also necessary that all the debaters be able to work with one another. Debaters Y & A cant work together. Debaters Z & C cant work together. Debaters A & B cant work together. If debater B is selected and debater Y is rejected, who are the members the team will consist of? If debater A is in the team, which other debaters must be in the team as well? If Y & Z are selected, which of the debaters must be in the team with them? 5. In a college a new football team was formed consisting of five members, 3 from Junior and 2 from Senior. In senior team the members are P, Q, R, S, in junior team the members are A, B, C and D. The selection of the team is based on the following conditions: P does not go along with R Q and R work together If B is selected, S is not selected C and D are always together. If P is selected the remaining members are: a) ACDS b) RBAB c) CDAQ d) QRBA 6. From amongst five doctors A, B, C, D and E, four engineers G, H, K and L and six teachers M, N, O, P, Q and R some teams are to be selected. Of these A, B, G, H, O, P and Q are females and the rest are males. The formation of teams is subjected to the following conditions: Wherever there is a male doctor, there will be no female teacher. Wherever there is a male engineer, there will be no female doctor. There should not be more than two male teachers in any team. If the team consists of two doctors, three female teachers and two engineers,who are the members of the team ? a) A B O P Q G H b) C D K L O P Q c) C D O P Q G H d) D E G H O P Q If the team consists of two doctors, one engineer and four teachers, all the following teams are Possible except? a) A B G M N O P b) A B H M O P Q c) A B H M R P Q Pondicherry Engineering College 58

Training & Placement d) A B K N R P Q If the team consists of two doctors, two female teachers, and two engineers, all the following teams are possible except : a) A B G H O Q b) A B G H P Q c) A B K L P Q d) O P G H A B 7. A team of five is to be selected from amongst five Boys P, Q, R, S and T and four Girls A, B, C and D. Some criteria for selection are: P and D have to be together. A cannot be put with C. S and B cannot go together. R and T have to be together. C cannot be put with Q. Unless otherwise stated, these criteria are applicable to all the questions below: If the two of the members have to be Boys, who will be the members of the team? a) P Q D A B b) P S D B C c) Q S D C B d) R T D A B If R be one of the members, the other members of the team are? e) A D P S f) BDPS g) B D R T h) D P R T 8. A Company wanted to start a new project. There was a need of five member team of 3 Managers and 2 Executives. The managers (A, B, C, D, E) and Executives (W, X, Y, Z) were to be selected with the following conditions: A and C work together X and Y work together but X does not work with Z H does not work with X; C and D work together If H is selected then A cannot be selected. If B is selected then who are the remaining Persons? 9. A school decided to go for NSS camp for a week. Boys team consists of A, B, C, D, E and Girls team consists of M, N, O, P. Six member team of 3 Girls and 3 Boys were to be selected with the conditions as A and B cannot be together M and P cannot be together P and B go together D and E go together M and N go together

Pondicherry Engineering College

59

Training & Placement 10. In a class there are 5 Boys A, B, C, D, E and 5 Girls L, M, N, O, P were to be selected for a competition. The selected team consists of 3 Boys and 2 Girls with the conditions as: From Boys side B and Girls side M should be selected A does not go with B A and E go together If B is not selected then L is selected M does not go with O N and P do not go together How many arrangements are possible?

3. Logical Reasoning
Concept: Logic: Our mind always seeks a rational explanation of the objects which we come in to contact with our day-to-day life. The perception and conception of the objects or elements become knowledge for us if our mind conceives them in a systematic order so that certain relationships can be established between the objects or elements. Knowledge of a subject is said to be systematic and scientific when the different parts of knowledge are related together in a particular way to make a system. This particular way of relationship between the elements, objects or subjects leads our mind towards the formation of thoughts which are expressed in language. This process is known as logic. So logic can be defined as the science of thought. Reasoning: When a thought is associated with the three attributes of What, Why and How its validity can be judged. These three attributes are often termed as reasoning. Reasoning is the function of the mind passing from known to unknown by establishing a systematic relationship between the elements.

3.1. Family Tree


3.1.1. Concept: Deals with Hierarchical structure of a family. It is also called as Blood relation problem. Problem 1: There are six members in a family A, B, C, D, E, and F who are related to each other. B is Fs daughterinlaw. D is As only grand child. C is Ds only uncle. A has only 2 children F and C, one male and one female. E is father of C. Then, 1.Who is the grand mother of D? 2.How many males and females are present? Solution: Denotes Female Denotes Male I. B is Fs daughter-in-law F Father/Mother-in-law

Daughter-in-law 60

Pondicherry Engineering College

Training & Placement II. D is As only grand daughter A

D III. C is Ds only uncle A

C C

IV. A has only 2 children F&C, one male and one female A

V. E is the father of C B A x E

Daughter-in-law

1. A is the grand mother of D. 2. There are 3 males and 3 females in the family.

Pondicherry Engineering College

61

Training & Placement 3.1.2. Problems to Solve: 1. 2. B is Brother of D. D is sister of E. E is brother of F. How is F related to B? Vivek is the father of M and Q. M is the son and Q is the daughter. L is the spouse of one of his child. N is of the same sex as Q. N and his/her spouse W, have two children S and R who are of the same sex as W. R and L are of different sexes. a) Who is the father of S and R? b) Whose spouse is L? c) If all the above characters belong to one family, how many males are there d) Who is the daughterinlaw of Vivek ? e) If N is Viveks sister, who is Ss uncle? A man was going with a Girl. Somebody asked his relationship with the Girl. He replied, My paternal uncle is the paternal uncle of her paternal uncles. What is the relationship between the man and the Girl? Out of A, B, C, D, E and F, C is sister of F. B is brother of Es husband. D has no brothers but is the father of A (male) and grandfather of F. We have 2 fathers, 3 brothers, one mother and 1 daughter. a) Find which are the brothers? b) Who is Es husband? Mohan is son of Aruns fathers sister. Prakash is son of Rekha who is mother of Vikki and grandmother of Arun. Purab is father of Neha and grandfather of Mona. Rekha is Purabs wife. How is Mohan related to Rekha? a) Grandson b) Son c) Nephew How is Vikki s wife related to Neha? a) Sister b) Niece c) Sister in law 6.

3.

4.

5.

d) Data Inadequate d) None

Looking at a portrait of a man, Sanjay said, His mother is the wife of my fathers son. Brothers and sisters I have none. At whose portrait was Sanjay looking? If Bs mother was As mothers daughter. How was A related to B? Introducing a lady, a man said, Her mother is the only daughter of my motherin law. What is the man to the lady? Fs grandfather is A. A has two sons E and C. C is married. How is F related to E?

7. 8.

9.

10. Anil introduces Rohit as, the son of the only brother of his fathers wife. How is Rohit related to Anil?

Pondicherry Engineering College

62

Training & Placement

3.2. Codes
3.2.1. Concept: Coding is a method of transmitting a message from one place to another. Decoding is the ability to decipher a certain code. In these types of questions, certain code values are assigned to a word or a group of words and the original words should be found out. Codes may be numerical or alphabets. It is easy to decode if we are able to find the format of arrangement. The arrangement can be any one of the following: 1. Reversed order 2. Gaps in between 3. +1 and1 format 4. Tree format

Problem 1: The term FIRE is coded as DGPC, then how will SNOW be coded as? Solution: The term FIRE is coded as DGPC, if we note it we find that the code is two alphabets preceding the original word. That is D E F, G H I, P Q R, C D E (Gaps in Between arrangement) So SNOW can be coded as QLMU. Problem 2: If NUMERICAL is written as MVLFQJBBK, then how. ASTROLOGY would be written in this code? Solution: In this Code, the letters in Odd place, i.e. in first, third, fifth, seventh and ninth place letters have been coded to their preceding letters and the remaining ones have next letters as code. Hence, The answer is ZTSSNMNHX. 3.2.2. Problems to Solve: 1. If in a certain language TEACHER is coded as VGCEJGT, how would DULLARD be coded in that code? 2. In a certain code, TRIPPLE is written as SQHOOKD How would DISPOSE be written in that code?

Pondicherry Engineering College

63

Training & Placement 3. In a certain code COMPUTER is written as RFUVQNPC How would MEDICINE be is coded as 4. If in a certain language CARROM is written as BZQQNL. How would HOUSE be written in that code language? 5. In a certain code RIPPLE is written as 613382 and LIFE as 8192. How should PILLER be written in that code? 6. In a certain code, PALE is written as 2134 and EARTH as 41590. How will PEARL be written in that code? 7. In a certain code 15789 is written as EGKPT and 2346 is written as ALUR. How is 23549 written in that code? 8. If room is called bed, bed is called window, window is called flower, flower is called cooler, then on what would a man sleep? 9. If cloud is called white, white is called rain, rain is called green, green is called air, air is called blue, blue is called Water, Where will the bird fly? 10. If in a certain language, CALCUTTA is coded as GEPGYXXE, Which word would be coded as FSQFCE?

3.3. Conditionality and Grouping


3.3.1. Concept: Conditions are given based on which the actions take place. There are four basic conditions. Based on them the decisions are made. Based on these conditions the groups are formed. Condition 1: If A occurs then B also will occur. Condition 2: If A occurs then B will not occur. Condition 3: If A has not occurred then B will occur. Condition 4: If A has not occurred then B also will not occur. Problem 1: From a group of 6 Boys A,B,C,D,E,F and 5 Girls L,M,N,O,P a group of 6 is to be selected based on the following conditions: O and P have to be together C cannot go with O Pondicherry Engineering College 64

Training & Placement A and D have to be together D cannot go with L C and M have to be together Band N have to be together B and E cannot be together If the team consists of four Girls the members of the team are: i) BELNOP ii) EFLNOP iii) BFLNOP If the team consist of 5 Boys and 1 Girl that Girl would be: i) L ii) M iii) N iv) O Solution: O and P will be there so C cannot be there as well as M. The Girls are L, N, O and P D and A cannot be there. So choice is (iii) BFLNOP If B is selected 2 Girls M and N have to be selected. So B is rejected. Therefore M is selected. 3.3.2. Problems to Solve: 1. In a college a new football team was formed consisting of five members, 3 from Junior and 2 from Senior. In senior team the members are A, B, C and D. In junior team the members are P, Q, R, S. The selection of the team is based on the following conditions: A does not go along with C B and C work together If Q is selected, D is not selected R and S are always together. If A is selected the remaining members are: (a) PRSD (b) CBPR (c) RSPB (d) BCQP If B and C are selected, how many arrangements are possible? 2. A school decided to go for NSS camp for a week. Boys team consists of P, Q, R, S, T and Girls team consists of W, X, Y, Z. Six member team of 3 Girls and 3 Boys was to be selected with the conditions as: P and Q cannot be together W and Z cannot be together Z and Q go together S and T go together W and X go together 3. From among 6 Boys A, B, C, D, E, and F and Five Girls P,Q,R,S and T a team of six is to be selected under the following conditions: A and D have to be together C cannot go with S S and T have to be together B cannot be teamed with E Pondicherry Engineering College 65

Training & Placement D cannot go with P B and R have to be together C and Q have to be together If four members have to be Girls, then find the members of the team? 4. A college selection committee sat to finalize Cricket Team of five members 2 from team X and 3 from team Z. Team X consist of 4 members A, C, E, G and Z team consist of 5 members B, D, F, H, J. The conditions given were: A and E go together but cannot go with B D and F go together but cannot play with H H and J go together but cannot play with E E does not play with D 5. In a class, 5 Boys A, B, C, D, E and 5 Girls L, M, N, O, and P were to be selected for a competition. The selected team consist of 3 Boys and 2 Girls with the conditions as, From Boys side B and Girls side M should be selected A does not go with B A and E go together If B is not selected then L is selected M does not go with O N and P do not go together How many arrangements are possible? 6. Eight students A, B, C, D, E, F, G and H are planning to enjoy car racing. There are only two cars and following are the conditions One car can accommodate maximum five and minimum four students. A will sit in the same car in which D is sitting and H is not in the same car. B and C cant sit in the same car in which D is sitting F will sit in the car of four people only along with A and E but certainly not with G. If H and G are sitting in the same car, who are the other two students sitting in the same car? (a) B and C (b) C and D (c) B and D (d) E and B (e) None of these If E and A are sitting in the same car, which of the following statements is true? (a) Five students are sitting in the same car (b) B is sitting in the same car (c) F is not sitting in the same car (d) G is not sitting in the same car (e) None of these Which of the following statement is superfluous for the above seating arrangements? (a) Only (i) (b) Only (ii) (c) Only (iii) Pondicherry Engineering College 66

Training & Placement (d) Only (IV) (e) None of these 7. At an Electronic Data Processing Unit, five out of the eight problem sets P, Q, R, S, T, U, V and W are to be operated daily. On any one day, except for the first day of a month, only three of the program sets must be the ones that were operated on the previous day. The program operating must also satisfy the following conditions: If Program P is to be operated on a day, V cannot be operated on that day. If Q is to be operated on a day, T must be one of the programs to be operated after Q. If R is to be operated on a day, V must be one of the programs to be operated after R. The last program to be operated on any day must be either S or U. Which of the following could be the set of programs to be operated on the first day of a month? (a) V, Q, R, T, S (b) U, Q, S, T, W (c) T, U, R, V, S (d) Q, S, R, V, U (e) P, R, V, S, U Which of the following is true of any days valid program set operation? (a) P cannot be operated at third place (b) Q cannot be operated at third place (c) R cannot be operated at fourth place (d) T cannot be operated at third place (e) U cannot be operated at fourth place If R is operated at third place in a sequence, which of the following cannot be the second program in that sequence? (a) S (b) T (c) U (d) W If the program sets R and W are to be operated on the first day, which of the following could be the other programs on that day? (a) P, T, V (b) Q, S, V (c) Q, T, V (d) T, S, U (e) T, S, V 8. A team of five is to be selected from amongst five Boys C, D and E and four Girls P, Q, R and S Some criteria for selection are: A and S have to be together. P cannot be put with R. D and Q cannot go together. Pondicherry Engineering College 67 A, B,

Training & Placement C and E have to be together. R cannot be put with B. Unless otherwise stated, these criteria are applicable to all the questions below: If the two of the members have to be Boys, the team will consist of? a) A B S P Q b) A D S Q R c) B D S R Q d) C E S P Q If R be one of the members, the other members of the team are? e) P S A D f) Q S A D g) Q S C E h) S A C E If two of the members are Girls and D is one of the members, the members of the team other than D are? i) P Q B C j) P Q C E k) P S A B l) P S C E If A and C are members, the other members of the team cannot be? m) B E S n) D E S o) E S P p) P Q E 9. From amongst five doctors A, B, C, D and E, four engineers G, H, K and L and six teachers M, N, O, P, Q and R, some teams are to be selected. Of these A, B, G, H, O, P and Q are females and the rest are males. The formation of teams is subject to the following conditions: Wherever there is a male doctor, there will be no female teacher. Wherever there is a male engineer, there will be no female doctor. There shall not be more than two male teachers in any team. If the team consists of two doctors, three female teachers and two engineers, the members of the team are? (a) (b) (c) (d) ABOPQGH CDKLOPQ CDOPQGH DEGHOPQ

If the team consists of two doctors, one engineer and four teachers, all the following teams are possible except: (a) ABGMNOP Pondicherry Engineering College 68

Training & Placement (b) (c) (d) ABHMOPQ ABHMRPQ ABKNRPQ

If the team consists of two doctors, two female teachers, and two engineers, all the following teams are possible except : (a) (b) (c) (d) ABGHOQ ABGHPQ ABKLPQ OPGHAB

If the team consists of three doctors, two male engineers and two teachers, who could be the members of the team? (a) (b) (c) (d) ABCKLMR BCDKLNR CDEKLMN CDEKLPR

If the team consists of two doctors, two engineers and two teachers, all the following teams are possible except? (a) (b) (c) (d) ABGHOP ABGHMN CEKLNR CDKLOP

10. A causes B or C, but not both F occurs only if B occurs D occurs if B or C occurs E occurs only C occurs J occurs only if E or F occurs D causes G, H or both H occurs if E occurs G occurs if F occurs If B occurs which must occur? a) D b) D & G c) G & H

d) F & G

e) J

If J occurs which must have occurred? a) E b) either B or C c) both E, F above

d) Both B and C

e) None of the

Which may occur as a result of cause not Mentioned. 1) D 2) A 3) E a) 1 only b) 2 only c) 1 & 2 d) 2 & 3

e) 1, 2, 3

Pondicherry Engineering College

69

Training & Placement

3.4. Series
3.4.1. Concept: Series involves the numbers or letters that are arranged in a particular sequence. The sequence can be any of the following; 1. Ascending 2. Descending 3. Gap 4. Squares and cubes 5. +/ order series. Problem 1: Fill in the series 1, 9, 25, _, 81. Solution: As given in the problem it consists of Squares of digits with a digit left in between It is 12, 32, 52, _, 92 So the solution is 72 = 49. 3.4.2. Problems to Solve: 1. Y, W, U, S, Q, __? __? 2. A, C, F, H, ?, M 3. Z, X, V, T, R, __ ? ___ ? 4. A, D, G, J, M, __? 5. 1, 4, 9, 16, 25, __ ? 6. 1, 6, 13, 22, 33, ___ ? 7. 121, 225, 361, ___ ? 8. 0.5, 1.5, 4.5, 13.5, __? 9. 1, 2, 6, 24, __ ? 10. 5, 9, 17, 29, 45,__ ?

3.5. Direction Sense


3.5.1. Concept: These problems test our sense of Direction. The best way of solving these questions is to follow the instructions given in the question carefully and make a diagram accordingly with the help of which the question can be solved. Problem 1: A starts from his office and walks 3 km towards north. He then turns right and walks 2 km and then turns right and walks 5 km. He then turns right and walks 2 km and then again turns right and walks 2 km. In which direction is he from the starting point? a) South b) Northeast c) SouthEast d) He is at the starting point Pondicherry Engineering College 70

Training & Placement

3 5

Solution: He is at the starting point. Problem 2: Deepa moved a distance of 75 metres towards the north. She then turned to the left and walking for about 25 metres, turned left again and walked 80 metres. Finally, she turned to the right at an angle of 450. In which direction was she moving finally? a) North-east b) North-west c) South d) South-east e) South-west Solution: Deepa started from A, moved 75 m upto B, turned left and walked 25 m upto C. She then turned left again and moved 80 m upto D. Turning to the right at an angle of 450 , She was finally moving in the direction DE i.e., South-west.

Hence, the answer is (e). 3.5.2. Problems to Solve: 1. Arman walked towards east for a distance of 5 km, turned towards right and walked for a distance of 10 km. Then again turned to his right and walked 15 km. Next he turned to his left and walked 5 km. In which direction is he now with reference to his starting point? 2. Rajesh is standing to the west of Amir and north of Ruchir and Sathish is to the west of Ruchir and south of Salman. In which direction is Sathish with reference to Amir? 3. One day Kannan left home and cycled 10 km southwards, turned right and cycled 5 km and turned right and cycled 10 km and turned left and cycled 10 km. How many kilometers will he have to cycle to reach his home straight?

Pondicherry Engineering College

71

Training & Placement 4. A man leaves for his office from his house. He walks towards East. After moving a distance of 20 m, he turns towards South and walks 10 m. Then he walks 35 m towards the West and further 5 m towards the North. He then turns towards East and walks 15 m. What is the straight distance in meters between his initial and final positions? 5. Ram started walking towards South. He took a right turn after walking 10 metres. He again took a left turn walking 20 metres. Which direction is he facing now? 6. A walks southwards then right, then left and then right. In which direction is he from the starting point? 7. I go 5 km east, then turn right and go 8 km. Then I turn left and go 5 km, and then I turn left and go 8 km. At what distance am I from the starting point now? 8. If A is to the South of B and C is to the east of B, in what direction is A with respect to C? 9. The Door of my house faces East. I walk from my back door 100 m, then right and walk 100m and turn left and walk 50 m and reach a point X. In which direction is I from the starting point? 10. Gaurav walks 20 m towards North. He then turns left and walks 40 m. He again turns left and walks 20 m. Further, he moves 20 m after turning to the right. How far is he from his original position?

3.6. Statement Logics


3.6.1. Concept: The ability of Analysis and interpretation of data logically is tested in these problems. Types: 1) statement conclusions 2) statement assumptions 3) statement arguments Conclusions In each questions there is a statement and two conclusions. conclusions and give answers. a) If conclusion 1 follows b) If conclusion 2 follows c) If neither 1 nor 2 follows d) Both 1 and 2 follows Example: Statement: Quality has a Price tag. India is allocating lot of funds for education. Logically apply the

Pondicherry Engineering College

72

Training & Placement Conclusions: 1) Quality of education in India would improve soon 2) Funding alone can enhance Quality of education Arguments Each question given below has 1 statement and two arguments. We should decide which of the arguments is strong Give answer: a) If only 1 is strong b) If only 2 is strong c) If neither 1 or 2 is strong d) If both 1 and 2 is strong Answer (a) Example: Should Private sector be permitted to operate telephone services? Arguments: 1) Yes, they are operated in advanced western countries 2) No, It is risky to put them in private hands

Assumptions Each question contains 1 statement and 2 assumptions. Check the data and answer. Give answer: a) If only 1 implicit b) If only 2 implicit c) Both 1 and 2 implicit d) Neither 1 nor 2 implicit Answer: Example: Statement: The pen is mightier than the sword Assumptions: 1) The pen is made up of stronger metal than the sword 2) The power of the mind is much stronger than brute physical power Answer: (1) (a)

Pondicherry Engineering College

73

Training & Placement 3.6.2. Problems to Solve: Statement Conclusion a) Only conclusion I follow b) Only conclusion II follows c) Either I or II follows d) If neither I nor II follows e) If both I and II follows 1. Statement: Morning walks are good for health. Conclusion: I) all healthy people go for morning walks. II) Evening walks are harmful.

2. Statement: Company X has marketed the Product. Go ahead; purchase it if Price and quality are your considerations. Conclusions: I) The Products must be good in quality. II) Then Price of the Product must be reasonable.

3. Statement: The Best way to escape from a problem is to solve it Conclusion: I) Your life will be dull if you dont face a problem. II) To escape from problems you should have solutions

4. Statements: Vegetable Prices are soaring in the Market Conclusions: I) Vegetables are becoming a rare commodity II) People cannot eat Vegetable Statement Argument a) If only 1 is strong b) If only 2 is strong c) If either 1 or 2 is strong d) If neither 1 nor 2 is strong e) If both 1 and 2 is strong 5. Statements: Should there be internal assessment in colleges? Argument: 1. Yes, it will enable the teachers to have a better control over the Students. 2. No, since it will encourage favoritism among teachers. 6. Statements: Should military service be made compulsory in our country? Argument: 1) No. It is against the policy of non violence. 2) Yes. Every citizen should protect his country. 7. Statements: Should there be world Government? Argument: 1) Yes. It will help in eliminating tensions among the nations Pondicherry Engineering College 74

Training & Placement 2) No. Then only the developed countries will dominate the Government Statement Assumption a) If only 1 implicit b) If only 2 implicit c) Either 1 or 2 implicit d) Neither 1 nor 2 implicit e) Both 1 and 2 implicit 8. Statements: To keep myself up to date , I always listen to 9.00 pm news on radio candidate tells the interview board. Assumption: 1) The candidate doesnt read newspaper. 2) Recent news is broadcasted only on radio

9. Statements: Double your money in five months An advertisement. Assumption: 1) The Assurance is not genuine. 2) People want their money to grow. 10. Statements: I cannot contact you on phone. Assumption: 1) Telephone facility is not available. 2) Now a Days it is difficult to contact on phone

4. Verbal Reasoning
Concept: It is the general notion that English is a foreign language and so it is very difficult. But this notion is far from truth. A student must understand that English is not a tough language. A language is a practical medium of expression of our ideas and thoughts. People who study and use a language are mainly interested in how they can do things with the language, how they can make meanings, get attention to their problems and interests, influence their friends and colleagues and create a rich social life for themselves. They are only interested in the grammatical structure of the language as a means of getting things done. Grammar puts together the patterns of the languages.Acquiring a simple, direct and forceful knowledge in writing and reading calls for constant practice. The questions in English Language are designed to test candidates understanding of English and its usage. The aim is to find how well candidates do understand the grammatical rules and their usages, how well they do comprehend the idea conveyed in the given passage and how well candidates can express the given idea. Verbal Ability is to understand the prose and to work with specialized and Technical Vocabulary. Verbal Ability falls under following categories: Pondicherry Engineering College 75

Training & Placement Synonyms Antonyms Analogy Sentence Completions Idioms Spotting errors Reading Comprehension

4.1 Antonyms
4.1.1 .Concept: The word Antonym means a word that is opposite in meaning to another word. For example, The Antonym for the word Old is New. Tips: 1. Before you look at the answer choices, try to clearly define the given word. Remember that you are looking for the answer choice that has a meaning opposite to that of a given word. 2. Consider slight variations in the meaning of each word. 3. If you have trouble with a definition, try using the word in the sentence. Try to predict an answer before looking at the answer choices. If an answer choice matches your predicted answer, it is most likely correct. 4. Use the Latin roots, prefixes, and suffixes to figure out what hard words mean. Look for cognates from French, Spanish, or Italian if you recognize them. 5. Always consider all the answer choices before you select an answer. Example: MAGNIFY a) Forgive b) Comprehend c) Extract d) Diminish e) Electrify Solution: Magnify (enlarge, expand) a) Forgive ( excuse, absolve) b) Comprehend ( understand, know, realize) c) Extract ( take out, remove, pullout) d) Diminish (reduce, make smaller) e) Electrify ( thrill, excite, amaze) The answer is Diminish. Pondicherry Engineering College 76

Training & Placement 4.1.2 Questions to Practice: 1. ABATE a) Aggravate b) Amplify c) Enlarge d) Enhance 2. BUCOLIC a) Urban b) City like c) Slovenly d) Polished

3. CATACLYSM a) Steadfastness b) Privilege c) Benefit d) Blessing 4. DEPRECATE a) Praise b) Approve c) Eulogize d) Depreciate 5. EMOLUMENT a) Penalty b) Punishment c) Monument d) Retribution 6. FRACTIOUS a) Complaisant b) Genial c) Sagacious d) Accommodating 7. ILLUSORY a) True b) Elusive c) Real d) Factual 8. PROVIDENTIAL a) Illstarred b) Unfortunate c) Unlucky d) Simulation Pondicherry Engineering College 77

Training & Placement 9. TURBULENT a) Quiet b) Quiescent c) Calm d) Truculent 10. VENAL a) Honourable b) Likeable c) Incorruptible d) Patriotic

4.2 Synonyms:
4.2.1 Concept: The word Synonym means a word or phrase with the same or nearly the same meaning as another in the same language. For example, Shut and Close are synonyms. Example: ASTUTE a) Sheer b) Noisy c) Astral d) Unusual e) Clever Astute: (smart, intelligent, shrewd) Sheer: (pure, absolute, complete) Noisy: (loud, piercing,) Astral: (planetary, lunar) Unusual: (strange, Odd, curious) Clever: (smart, bright, intelligent) Answer Clever 4.2.2 Questions to Practice: 1. VARIEGATE a) Set type b) Multicolour c) Differ d) Reject e) Reply in Kind 2. FILCH a) Pretend b) Dirty c) Embarrass d) Steal e) Honour

Pondicherry Engineering College

78

Training & Placement 3. INFINITE a) Verbal b) Indefinite c) Endless d) Strange e) Vague 4. DEMISE a) Residence b) Dismissal c) Accident d) Act e) Death 5. FRUGALITY a) Extravagance b) Ripening c) Thrift d) Miserliness e) recurrent 6. UNEQUALED a) Outstanding b) Different c) Praised d) Resentment e) Miserliness 7. ADVERSITY a) Opponent b) Hardship c) Opening d) Public announcement e) Agency 8. FASTIDIOUS a) Speedy b) Precise c) Squeamish d) Hungry e) Slow 9. DISCONCERT a) Sing in harmony b) Pretend c) Cancel Program d) Confuse e) Interrupt 10. GARRULOUS Pondicherry Engineering College 79

Training & Placement a) b) c) d) e) Laconic Strangling Ecstatic Frozen Wordy

4.3 Analogy
4.3.1 Concept: Analogy questions ask you to determine the relationship between the two words in a pair and then to recognize a similar or parallel relationship between the members of a different pair of words. The relationship between the words in the original pair will always be specific and precise. Tactics: State the relationship between the capitalized words in a clear sentence. If there is more than one answer, look for a narrower approach Consider secondary Meaning as well primary Meaning. Familiarize yourself with common analogy types. 4.3.2 Analogy Types 1. Definition 2. Defining Characteristic 3. Class and members 4. Antonyms 5. Synonyms 6. Part to Whole 7. Function 8. Manner 9. Action & its significance 10. Worker and article created 11. Worker and tool 12. Worker and action 13. Sex 14. Symbol Example Book: Library:: Cannon: a) Artillery b) Powder c) Shell d) War Solution: Library is the place to keep books. Cannon Means gun so Artillery is the place to keep gun, mortar 4.3.3 Questions to Answer: Pondicherry Engineering College 80

Training & Placement 1. EXPLOSION: DESTRUCTION (a) Talk (b) Girl (c) Success (d) Engagement 2. AGENDA: MEETING (a) Program (b) Performance (c) Map (d) Foot note 3. CURTAIN: DRAPERY (a) Cockroach (b) Bed sheet (c) Pillow (d) Mat 4. BALANCE: WEIGH (a) Airplane (b) Radar (c) Satellite (d) Television 5. CROWN: ROYAL (a) Throne (b) Wrap (c) Pen (d) Crucifix 6. REVOLVER: HOLSTER (a) Book (b) Eye (c) Juice (d) Nostril 7. SYMPHONY: MUSIC (a) Mural (b) Ode (c) Preface (d) Editorial 8. TRAITOR: DISLOYALTY (a) Executioner (b) Rebel (c) Manager (d) Hope 9. TEXTILE: MILL (a)Eggs ` (b)Coal (c)Food (d)Brick

: : : :

Exaggeration Woman Failure Marriage

: : : : : : : : : : : : : : : : : : : : : : : : : : : : : : : :

Function Ticket Scale Article Insect Bed Cushion Floor Height Detection Revolution Picture Regal Ermine Author Religion Bag Eyelid Glass Nose Painting Prose Book Journal Reliability Defiance Administration Pessimism Hen Mine Agriculture Kiln 81

Pondicherry Engineering College

Training & Placement 10. MODESTY: ARROGANCE (a) (b) (c) (d) Passion Practice Cause Debility : : : : Emotion Perfection Purpose Strength

4.4. Sentence Completion


4.4.1 Concept: It tests your a) b) Ability to use vocabulary Ability to recognize logical consistency among the elements in a sentence

What makes Sentence Completion difficult is A. Vocabulary: Use of words like nonplussed, Harbingers, Eclectic.

Example: Economy restrain Toy Dally B. Grammatical complexity : They include entire range of grammatical possibilities. Use of clauses, prepositional phrases, gerunds etc. in a convoluted manner C. Tone: Writers attitude towards the subject matter i.e. ironic, playful, skeptical, somber etc. D. Style: Ideas maybe expressed in different manners, ornately on sparely, poetically or prosaically formally or informally etc. An authors style depends on such details as word choice, imagery repetition, rhythm, sentence structures and length. Thought extenders Continue the idea Thought contrasters Reverse the idea Signal Words: Additionally Also And As well Besides Further more Although But Despite nevertheless In spite of On the contrary Pondicherry Engineering College 82

Training & Placement More over Still Too While Likewise Which Tactics: Read the sentence and think of a word that makes sense. Look at all possible answers. In double blank, go through the answer, testing the first word in each choice. Watch for single words that link one part of the sentence to another. Break down complex sentences into simpler components. Check whether the Metaphor controls the choice of words. Example: Because experience had convinced her that he was both selfseeking and avaricious, she rejected the likelihood that his donation had been ______. a) Redundant b) Frivolous c) Inexpensive d) Ephemeral e) Altruistic Solution: The sentence presents simple case of cause and effect. Opposite of self seeking (selfishness), Avaricious (Greediness) Altruistic (unselfish, selfless) 4.4.2 Questions to Answer: 1. In one shocking instance of _______ research, one of the nations most influential researchers in the field of genetics reported on experiments that were never carried out and published deliberately ________ scientific papers on his nonexistent work. (a) Comprehensive.. abstract (b) theoretical challenging (c) fraudulent deceptive (d) Derivative .. authoritative (e) Erroneous . Impartial 2. Measurement is, like any other human Endeavour, a complex activity, subject to error, not always used ________ and frequently misinterpreted and _________. (a) Mistakenly .. decided (b) erratically analyzed (c) Systematically . Organized (d) Innovatively .. refined (e) Properly .. misunderstood 3. In a revolutionary development in technology, several manufacturers now make biodegradable forms of plastic; some plastic sixpack rings, for example, gradually ________ when exposed to sunlight. (a) Harden (b) Astagnate Pondicherry Engineering College 83

Training & Placement (c) (d) (e) Inflate Propagate Decompose

4. To alleviate the problem of contaminated chicken, the study panel recommends that the federal government shift its inspection emphasis from cursory birdbird visual checks to a more _______ random sampling for bacterial and chemical contamination. (a) Rigorous (b) Perfunctory (c) Symbolic (d) Discreet (e) Dubious 5. The Chinese, who began systematic astronomical and weather observations shortly after the ancient Egyptians, were assiduous recordkeepers, and because of this, can claim humanitys longest continuous _______ of natural events. (a) Refinance (b) Documentation (c) Maintenance (d) Domination 6. An institution concerned about its reputation is at the mercy of its members, because the misdeeds of individuals are often used to _______ the institutions of which they are a part. (a) Reform (b) Discredit (c) Coerce (d) Intimidate 7. Job failure Means being fired off a job, being asked to resign, or leaving _______ to protect yourself because you had very strong evidence that one of the first two was _______ (a) Voluntarilyimpending (b) Abruptlysignificant (c) Knowinglyoperative (d) Eventuallyintentional 8. During the widespread fuel shortage, the Price of gasoline was so _______ that suppliers were generally, thought to be _______ the consumer. (a) Reactiveshielding (b) Stableblackmailing (c) Excessivegouging (d) Depressedcheating 9. Normally an individual thunderstorm lasts about 45 minutes, but during certain conditions the storm may _________, becoming ever more severe, for as long as four Hours. (a) Wane (b) Moderate Pondicherry Engineering College 84

Training & Placement (c) (d) (e) Persist Vacillate Disperse

10. Perhaps because something in us instinctively distrusts such displays of natural fluency, some readers approach John Updikes fiction with __________. (a) Indifference (b) Suspicion (c) Veneration (d) Recklessness (e) Bewilderment

4.5 Spotting Errors


4.5.1 Concept: Grammatical errors form the most probable errors in error-spotting exercises. Mistakes in preposition and tenses are more common Examples: 1) The car is / almost / the same / like mine. Ans: as mine 2) The weather / feels / as / spring Ans: like spring 4.5.2 Questions to Answer: 1. Ashoka the Great / was regarded one/ of the greatest emperor the/ world has ever A B C D Produced / No error E 2. None of the two Boys / who were / present there / came to his help. / No error. A B C D E 3. More we get, more we want/ in this way/ there is no end to human desires/ No error. A B C D 4. When I arrived / on the station / I saw a man who had robbed me / because he A B C thought I carried the Kings wallet. / No error D E 5. He had ordered you/ to paint it green/ but you painted/ the blue house. / No error. A B C D E 6. Little money/ that he was left behind after/ the marriage of his daughter/ was not A B C Sufficient for a big family like his./ No error. D E Pondicherry Engineering College 85

Training & Placement 7. No less than/ four thousand people/ lost their lives/ in the earthquake/ no error. A B C D E 8. He wished me/ to dine with him that evening/ but which/ I declined/ No error. A B C D E 9. I have read several plays of Shakespeare/ who was one of / the greatest dramatists/ A B C the world has ever produced/ No error. D E 10. Nobody in their senses/ would have/ uttered/ such silly remarks/ No error. A B C D E

4.6. Idioms
4.6.1 Concept: A Group of words whose meaning is different from the meanings of the individual words are idioms. For example, Catch in the act means Finding someone doing something wrong or bad. Example: The managers bark is worse than his bite a) He shouts b) He punishes c) His action is worse than his speech d) His speech is worse than action Solution: The managers bark is worse than his bite d) His speech is worse than action Questions to Answer: 1. (a) (b) (c) (d) 2. (a) (b) (c) (d) Mary thought that the test was a piece of cake. Test was about eating a cake. She ate cake before the test. The task was relatively easy. She was thinking of eating a cake. Im trying to sell off these books; I dont want them on my hands any longer. The books are heavy. I will not carry the books. He has paid for the books. He wants to get rid of the responsibility.

3. During our winter break, my friends and I hit the slopes. (a) Hit each other. Pondicherry Engineering College 86

Training & Placement (b) Went snowskiing (c) Fell in the slopes. (d) Hit the mountains. 4. He had climbed almost to the top of the rock, but lost his nerve and turned back. (a) Got scared (b) Fell down (c) Falls down from the rock (d) His nerve was cut. 5. In spite of many different jobs he had to do, Jack always managed to keep his eye On the ball (a) He is a cricket player (b) Being focused. (c) Hit in the eye with a ball. (d) He is searching the ball. 6. (a) (b) (c) (d) My father insisted that I put my nose to the grindstone next semester. Hit my nose if I fail Work really hard Grind the stone Copy during the exam

7. Throughout the Summer, I lived a stones throw from a popular beach. (a) Throwing stones and playing. (b) People threw stones at his house. (c) Stones were piled on the beach (d) Lived near the beach. 8. (a) (b) (c) (d) 9. (a) (b) (c) (d) I havent studied at all for the exam tomorrow; Im really going to be in soup. Drink soup before exam. Fall into the soup Get into trouble. Prepare soup Sandy is often too bogged down with her studies to spend time with her friends. Happy Overwhelmed Forgetful Smart

10. The Police found the case a tough nut to crack. (a) Police could not crack nuts. (b) The case was difficult to solve. (c) Police found some nuts in the crime scene. (d) The case was easy to solve.

Pondicherry Engineering College

87

Training & Placement

4.7. Reading Comprehension


4.7.1 Concept: Questions on reasoning comprehension resemble questions set on English Comprehension. The only difference is that questions are designed to evaluate the Reasoning Power of the candidates. The Questions are generally of the following types: 1. 2. 3. 4. What is the assumption of the Passage? Which of the statements weakens the Argument? Which of the statements strengthens the Argument? Whether a Particular statement can be deduced from the passage or not.

What is an Assumption? Many statements are made by presuming something to be true. For example if we say, The Change in the Public Mood is Noticeable, it assumes that There has been a Change in the Public Mood. If we dont assume it, the statement becomes incorrect. Which statement Weakens or Strengthens the Argument? Any Statement that goes Contrary to the assumption or to the Supporting Argument weakens it. Any argument which gives ideas similar to the Supporting Argument strengthens it.

What is a Deduction? If we can infer a statement either by correlating information or by discovering an implication, it is said to be a deduction. Example: The emotional appeal of imperialism never completely stilled the British conscience. However, liberal thinkers throughout the nineteenth century argued that democracy was incompatible with the maintenance of authoritarian rule over foreign peoples. To think imperially was to think in terms of restrictive and protective measures; in defiance of the revealed truths of classical economics. Thus when the British government took over responsibility for India from the East India Company in 1858, many politicians were conscious of saddling Britain with a heavy burden. In the first seventy years of the nineteenth century, enlightened British liberals looked forward to the day when India would stand on its own feet. Even in the heyday of colonialism British radicals continued to protest that self-proclaimed imperialists however honorable their motives, would place faith accomplishment before the country and commit blunders of incalculable consequence. The danger, they felt was all the greater because British foreign policy still remained a stronghold of the aristocracy, while that related and persuasive lobby, the British officer class, also had a vested interest in imperial expansion. It took the humiliation of the Boer war to teach the British government what it would cost to hold an empire by force. However this fact did not escape Gandhi, the supreme tactician of the Indian Pondicherry Engineering College 88

Training & Placement liberation movement. He saw what some perceptive British thinkers had much earlier recognized, namely, that Britain could not long continue to rule India expect with the co-operation of many sections of its population. Once that cooperation was withdrawn, the foundation of British authority in India would crumble. Furthermore, the Indian nationalist leaders were able to exploit the aversion of the British liberal conscience to methods used by the local colonial rulers in combating Indian non co-operation. 1. What does the term authoritarian rule mean? (a) Rule of the dictionary of law (b) Dictatorial rule of an aristocrat un-accomplished by the rule of law (c) Arbitrary exercise of power by officials (d) Rule having stability (e) None of these Answer : ( B) 2. What according to the author was the attitude of the British liberals towards the British imperialist and colonial policy? (a) One of active co-operation (b) One of only verbal co-operation (c) One of total indifference (d) One of repeated protests (e) One of disagreement Answer : ( D)

3. Who was the supreme tactician of the Indian liberation movement? (a) Mrs. Annie Besant (b) The enlightened British liberals themselves (c) Lokmanya Balgangadhar Tilak (d) Mahatama Gandhi (e) None of these Answer : ( D) 4. What according to you would be the most suitable title for this passage? (a) British imperialism and India (b) British liberals attitude towards imperialism (c) Role of mahatma Gandhi in Indian freedom movement (d) The emotional appeal of British imperialism (e) British as a colonial power. Answer : ( D) 4.7.2 Question to Answer: Read the following passage and pick up correct answers for each of the questions which follow: Pondicherry Engineering College 89

Training & Placement Passage 1: A standard comprises characteristics attached to an aspect of a process or Product by which it can be evaluated. Standardization is the development and adoption of standards. When they are formulated, standards are not usually the Product of a single Person, but represent the thoughts and ideas of a group, leavened with the knowledge and information which are currently available .Standards which do not meet certain basic requirements become a hindrance rather than an aid to progress. Standards must not only be correct, accurate, and practice in requiring no more and no less than what is needed for satisfactory results, but they must also be workable in the sense that their usefulness is not nullified by external conditions. Standards should also be acceptable to the people who use them. If they are not acceptable, they cannot be considered to be satisfactory, although they may posses all the other essential characteristics. 1. According to the above paragraph, a processing standard that requires the use of materials that cannot be procured is most likely to be (a) Incomplete (b) Unworkable (c) Inaccurate (d) Unacceptable 2. According to the above paragraph, the construction of standards to which the performance of job duties should conform is most often. (a) Work of people responsible for seeing that the duties are properly performed. (b) Accomplished by the Person who is best informed about the functions (c) The responsibility of the people who are to apply them. (d) Attributable to the efforts of various informed Persons.

3. According to the above paragraph, when standards call for finer tolerances than those essential to the conduct of successful Production operations the effect of the standards on the improvement of Production operations. (a) Negative (b) Negligible (c) Nullified (d) Beneficial 4. Which one of the following is the most suitable title for the above paragraph? (a) The evaluation of formulated standard (b) The attributes of satisfactory standards (c) The adoption of acceptable standards (d) The use of process or Product standards. Passage 2: The chief condition of happiness, barring certain physical prerequisites, is the life of reasons the specific growth and power of man. Virtue, or rather excellence ,will depend on clear judgment , self control, symmetry of desire , artistry of Means; it is not the possession of the simple man, nor the gift of innocent intent, but the achievement of experience in the fully developed man. Yet there is a road to it, a guide to excellence, which may save many detours and delays: it is the middle way the golden Mean. The qualities of character can be arranged in triads in each of which they first and the last qualities will be extremes and vices, and the middle quality a virtue or an excellence. So between cowardice and rashness is courage: between stinginess and extravagance is liberality; between sloth and greed is ambition; Pondicherry Engineering College 90

Training & Placement between humility and pride is modesty; between secrecy and loquacity is honesty; between moroseness and buffoonery is good humour between quarrelsomeness and flattery is friendship; between Hamlets indecisiveness and Quixotes impulsiveness is self control Right, in ethics or contact is not different from right in mathematics or engineering ; it Means correct and fit what works best to get the best results. 1. What is the main idea of passing? (a) The qualities of character are there extremes and middle (b) In some respects ethics and mathematics resemble (c) Happiness can be achieved by following the middle path (d) None of these 2. What is the implied Meaning of the passage? (a) Happiness depends upon physical and mental qualities (b) Self control is necessary (c) Excellence should be achieved (d) Rational approach lies in following the middle path 3. The author has not said: (a) The middle path between humility and pride is modesty. (b) Middle path avoids delay in achieving excellence; right in ethics means that works to get the best results. (c) Courage is the middle path of indecisiveness and impulsiveness (d) None of the above.

4. Which of the following is not the middle path of different qualities? (a) Liberality (b) Ambition (c) Friendship (d) Secrecy 5. Who of the following is not the writer of either Hamlet or Don Quixote? (a) Ben Johnson (b) Shakespeare (c) Cervantes (d) None of these 6. Which of the following is the most suitable title for the passage? (a) Qualities of Character (b) Chief Condition of happiness (c) Golden Mean (d) None of these Passage 3: To a greater or lesser degree all the civilized communities of the modern world are made up of a small class of rulers, corrupted by too much power, and of a large class of subjects, corrupted by too much passive and irresponsible obedience. Participation in a social order of this kind makes it very difficult for individuals to achieve that non attachment in the midst of activity, which is the distinguishing mark of the ideally excellent human being; and where there is not at least the considerable degree of non attachment in activity, the ideal society of the Profits cannot be realized. A desirable social order is one that delivers us from avoidable evils. A bad social order is one that Pondicherry Engineering College 91

Training & Placement leads us into temptation which, if matters were more sensibly arranged, would never rise. Our present Business is to discover what large scale charges are best calculated to deliver us from the evils of too much power and off too much passive and irresponsible obedience. It has been shown that the economic reforms, so dear to advance thinkers are not in themselves Sufficient to produce desirable changes in the character of the society and of the individuals composing it unless carried out by the right sort of Means and in the right sort of governmental administrative and educational context, such reforms are either fruitless or actually fruitful evil. In order to create the proper contexts for economic reform, we must change our machinery of government, our methods of public administration, our system of education and our metaphysical and ethical beliefs 1. The main idea of the passage is (a) Ideal society is one which is free from evils. (b) Mans participation is necessary (c) Noattachment is the distinguishing mark. (d) Reforms are necessary for freeing us from power and obedience. 2. What is the inference that one derives? (a) Changes in the character of society are necessary (b) Ideal society is envisage by Profits (c) It is the mark of ideally excellent human beings (d) None of these. 3. The author does not say (a) By participating in this kind of society one cannot remain nonattached. (b) Subjects indulge in obedience (c) Bad social order leads to temptations (d) None of these. 4. Who do you think the writer is? (a) Capitalist (c) Anarchist 5. The passage implies (a) Power and obedience corrupt the society. (b) Power is of no significance. (c) Changes in society are natural. (d) Economics is necessary. (b) Socialist (d) None of these.

Pondicherry Engineering College

92

S-ar putea să vă placă și